SlideShare a Scribd company logo
1 of 27
Structures of skin
1) Dermal dendrocytes

A. are the primary cell found in a glomangioma.
B. actively synthesize and release IgG.
C. are responsible for immediate-type hypersensitivity reactions.
D. present antigen to naïve T cells in the lymph node. Correct Choice
E. are the primary cell found in an angiosarcoma.
A dermal dendrocyte is a mononuclear phagocytic cell that is a type of antigen presenting cell. It
is derived from the bone marrow and found both in the papillary and upper reticular dermis. This
cell is highly phagocytic and synonymous with the melanophage that has ingested pigment.
Dermal dendrocytes are likely very important to the afferent limb of the immune response.

2) What is the major component of the anchoring fibril?

A. Type III Collagen
B. Type I Collagen
C. Type VII CollagenCorrect Choice
D. Type IV Collagen
E. Laminin 5
Anchoring fibrils are made of Type VII collegen. Anchoring plaques are made of Type IV collagen,
and interact with a network of Type I And Type III collagen fibers in the dermis

3) What is the most abundant collagen found on fetal skin?

A. Type III CollagenCorrect Choice
B. Type II Collagen
C. Type IV Collagen
D. Type VII Collagen
E. Type I Collagen
Type III collagen is found in the fetal skin. It is also present in the gastrointestinal tract, blood
vessels, and the basement membrane. A defect in this collagen results in the Ehlers-Danlos,
vascular type.

4) Mutations in which of the following proteins results in epidermolysis bullosa simplex associated
with muscular dystrophy

A. Laminin 5
B. Uncein
C. collagen type IV
D. Plectin Correct Choice
E. a6b4 integrin



                                                    1
Anchoring filaments exist within the lamina lucida. They are primarily comprised of laminin 5 and
BP180. Laminin 5 is a cross-shaped assembly of 3 classes of polypeptides, a, b, g. The
anchoring filaments function as a structural network to which other proteins attach, and they
function as signaling molecules that transmit morphogenetic information to transmembrane
proteins of the basal cell layer (such as the integrins). Laminin 5 is also called epiligrin and binds
to the a6b4 integrin at the hemidesmosome.

5) The finding on DIF that reflects binding of the Ro and La antigens in subacute cutaneous lupus
erythematosus is:

A. Immune deposits along the DE junction
B. Granular deposits along the basement membrane
C. Cytoid bodies
D. Granular fluorescence throughout the cytoplasm and nucleus of basal keratinocytesCorrect
Choice
E. A "chicken-wire" pattern within the epidermis
Granular fluorescence throughout the cytoplasm and nucleus of basal keratinocytes reflect the
binding of Ro and La antigens and is unique to SCLE. Cytoid bodies and Immune deposits along
dermal-epidermal junction are seen in both DLE and SCLE. Granular deposits along the
basement membrane are seen in dermatitis herpetiformis and a chicken-wire pattern of staining is
seen in pemphigus vulgaris.

6) Which of the following markers are specific and reliable for Merkel cells?

A. CD34
B. CK20Correct Choice
C. CD20
D. HMB-45
E. CD3
Cytokeratin (CK) 20 is a reliable marker for Merkel cells. CD20 is a marker for B-cells and CD3 is
a T-cell marker. HMB-45 is used in staining for immature melanosomes and is reactive in
melanoma. CD34 is used to stain dermatofibrosarcoma protuberans.

7) Apocrine glands are found in all of the following areas of the body except:

A. Axillae
B. Perineum
C. Breasts
D. Eyelid
E. Palms Correct Choice
Apocrine glands operate by decapitation secretion and are activated by epinephrine and
norepinephrine. They are located in a few distinct areas of the body, which include axillae,
anogenital region, Moll’s glands of the eyelids, mammary glands of the breast and the
ceruminous glands of the external auditory canal.

8) Which mechanoreceptor found in hair bearing areas sense deep touch and vibration?



                                                   2
A. Krase end-bulb
B. Vater-Pacini corpuscleCorrect Choice
C. Merkel cell
D. Meissner corpuscle
E. Free nerve ending
Vater-Pacini corpuscles sense deep touch and vibration. Merkel cells are slow adapting type I
mechanoreceptors found among basal keratinocytes. Meissner corpuscles are found in the
dermal papilla, especially in the palms and soles. Krase end-bulbs are mucocutaneous end-
organs found on the glans penis, prepuce, clitoris, labia minora, and vermillion border of the lip

9) Glomus cells are

A. monocytes.
B. vascular smooth muscle cells. Correct Choice
C. modified fibroblasts.
D. neuronal cells.
E. modified skeletal muscle cells.
Glomus cells are derived from Suquet-Hoyer canals. They are vascular smooth muscle cells that
allow the rapid shunting of blood from the arterioral to venular system, bypassing the capillaries.
This process occurs primarily on the palms and soles.

10) Which protein is the largest component of the cornified cell envelope?

A. Keratin
B. Profilaggrin
C. Involucrin
D. LoricrinCorrect Choice
E. Ceramide
The cornified cell envelope is a durable, protein-lipid polymer that eventually acts as a
mechanical and chemical barrier on the exterior of cornified cells. In the upper spinous layer,
keratohyaline granules release profilagrin and loricrin. Profilaggrin is cleaved to filaggrin and
subsequently aggregates keratin filaments. Loricrin is the major protein component of the
cornified cell envelope and is bound to the cell membrane by transglutaminases (in addition to
other structural proteins like involucrin, keratins, elafin, cystatin A and desmosomal peptides).
This forms the highly insoluble proteinaceous component of the cornified cell envelope.

11) Embryologically, epidermal stratification occurs at approximately what estimated gestational
age?

A. 20 weeks
B. 16 weeks
C. 8 weeksCorrect Choice
D. 12 weeks



                                                  3
E. 4 weeks
Epidermal stratification begins at about 8 weeks estimated gestational age and is completed by
the 2nd trimester. Epideral stratification occurs when an 'intermediate layer' develops between
the epidermal basal cells and the overlying layer of periderm cells. The periderm is an embryonic
structure that covers the primitive epidermis until keratinization begins, at which point the
periderm sloughs off and contributes to the protective covering of the newborn, the vernix
caseosa. This intermediate layer is highly proliferative, such that by 24-25 weeks estimated
gestaional age, the epidermis consists of 4 or 5 layers, in addition to the degenerating periderm

12) A specific marker of Merkel cells is

A. Envoplakin
B. Cytokeratin 10
C. Loricrin
D. Cytokeratin 20 Correct Choice
E. Cytokeratin 15
Cytokeratin 20 is a specific marker for the Merkel cell. Merkel cells are mechanoreceptors located
at body sites requiring high tactile sensitivity. Keratinocyte deformation results in a secretion of
chemokines by Merkel cells, which make synaptic connection with neurons.

13) On electron microscopy, which cell demonstrates cytoplasmic projections and secretory
granules?

A. macrophage
B. Langerhans cell
C. keratinocyte
D. mast cellCorrect Choice
E. melanocyte
Electron microscopy of mast cells demonstrates large long villi at the periphery and round or oval
secretory granules. Langerhans cells show a folded nucleus and rarely phagocytized
melanosomes on electron microscopy. Their characteristic feature is the presence of Birbeck
granules whose disk shape with one or two vesicles at either end represents a tennis raquet.
Examination of melanocytes shows an absence of tonofilaments or desmosomes with
characteristic melanosomes in various stages of formation. Macrophages often contain
phagocytized material within phagosomes.

(Lever's Histopathology of the Skin 8th ed 1997 p.32.)

14) Upon presentation of an antigen in the skin surface, a hapten forms. The first cell to take up
the hapten is

A. Mast cell
B. T cellsCorrect Choice
C. B cells
D. keratinocytes




                                                 4
E. Langerhans cells
Most of the contact allergens are low-molecular weight chemicals, which after penetrating into the
skin, have to couple with host proteins to be able to act as full antigens. These are called
haptens. Upon epicutaneous application to a naive host, Langerhans cells take up the hapten,
process it and migrate towards the regional lymph nodes, where the anitgen is presented to the
naive T cells

15) Elastic fibers contain the specific amino acids

A. Alanine and phenylalanine
B. Glycine and proline
C. Desmosine and isodesmosine Correct Choice
D. Leucine and isoleucine
E. Lysine and proline
Desmosine and isodesmosine are the typical amino acids of elastic fibers. Elastic fibers are
comprised of elastin that is wrapped by fibrillin microfibrils. Elastic fibers form a complex
meshwork extending from the lamina densa of the dermo-epidermo junction through the dermis.
Elastic fibers return the skin to a normal shape after being stretched.

16) Apocrine chromhidrosis results in from which of the following contents of apocrine sweat?

A. squalene
B. cholesterol
C. ammonia
D. lipofuschinCorrect Choice
E. fatty acids
Chromhidrosis refers to the secretion of pigmented sweat, most commonly yellow, green or black.
It reflects the rich lipofuschin content of apocrine sweat. Extrinsic apocrine chromhidrosis results
from staining of sweat and garments by chromogenic bacteria, such as Corynebacterium spp.

17) The major component of the anchoring filaments is

A. Uncein
B. Plectin
C. Laminin 5 Correct Choice
D. a6b4 integrin
E. Collagen type IV
Anchoring filaments exist within the lamina lucida. They are primarily comprised of laminin 5 and
BP180. Laminin 5 is a cross-shaped assembly of 3 classes of polypeptides, a, b, g. The
anchoring filaments function as a structural network to which other proteins attach, and they
function as signaling molecules that transmit morphogenetic information to transmembrane
proteins of the basal cell layer (such as the integrins). Laminin 5 is also called epiligrin and binds
to the a6b4 integrin at the hemidesmosome.

18) Which of the following make up the major protein of the cornified cell envelope?



                                                   5
A. Filaggrin
B. LoricrinCorrect Choice
C. Involucrin
D. Laminin V
E. Envoplakin
Loricrin is the major protein component of the cornified cell envelope (CE). Involucrin is cross-
linked by transglutaminae in the granular layer to form an insoluble cell boundary. Envoplakin
may link the CE to desmosomes and to kertin filaments. Filaggrin is thought to promote
aggregation and disulfide bonding of keratin filaments in CE. It is degraded into urocanic acid and
pyrrolidone carboxylic acid. Both of which hydrate the stratum corneum and block UV radiation.
Laminin V is found in the basement membrane and is not involved in the formation of the
cornified cell envelope.

19) Tissue contraction begins

A. after the 3rd month of wound healing.
B. after the first month of wound healing.
C. after the 9th month of wound healing.
D. at 3rd day of wound healing.
E. during the 2nd week of wound healing. Correct Choice
See discussion of wound healing from questions 29 through 33.

20) Telogen effluvium

A. results from sudden illness or surgery. Correct Choice
B. involves a diffuse alopecia affecting more than 50% of the scalp.
C. is an inflammatory alopecia devoid of scarring.
D. often results from antimetabolites used during cancer chemotherapy.
E. is a patchy alopecia affecting less than 50% of the scalp.
Telogen effluvium is an excessive loss of club hairs from the normal resting follicles of the scalp.
The follicle is pushed from anagen to catagen to telogen. There is no inflammatory process
involved. Causes of telogen effluvium are illness, surgery, parturition, fever, drugs, traction,
starvation, and hypervitaminosis A. Usually, the hair loss only involves less than 50% of the
scalp. There is no specific therapy, and most cases are self-resolving within months.

21) Cutaneous warts in renal transplant recipients show increased expression of which keratin?

A. K2
B. K16
C. K5
D. K9
E. K13Correct Choice




                                                  6
Keratins are divided into acidic (type I) and basic (type II) subtypes. Keratin 13 is a type I keratin
which is expressed suprabasally in adult epithelia and is associated with terminal differentiation.
Renal transplant recipients have an increased risk of developing skin cancers and multiple warts.
Factors contributing to this increased risk include human papillomavirus infection,
immunosuppressive therapy, and exposure to ultraviolet radiation. Increased expression of K13
has been demonstrated in high-risk papillomas in this patient population.

(Blokx W. Arch Dermatol 2002; 138:61-65

22) Which of the following glands is not under neural control?

A. Eccrine glands
B. Apocrine glands
C. Salivary glands
D. Sebaceous glandsCorrect Choice
E. Ceruminous glands
Sebum is secreted continuously on the skin under hormonal glands. It contains squalene,
cholesterol, cholesterol esters, wax, and triglycerides

23) The major protein component of the cornified envelope is

A. Plectin
B. Envoplakin
C. Loricrin Correct Choice
D. Transglutaminase
E. Desmoplakin
Loricrin is the major component of the cornified envelope (CE). The proteins of the CE are
synthesized in the spinous and granular layers.

The CE is primarily a protein/lipid polymer formed within the differentiating layer of keratinocytes.
The CE eventually exists outside of the cornified cells after the granular cell undergoes a
programmed destruction (apoptosis). Self-destructing granular cells are called transition cells.

24) Acid keratins (K10-20) are expressed on which of the following chromosomes?

A. 12
B. 17Correct Choice
C. 18
D. 19
E. 10
Acid keratins are expressed on chromosome 17 and basic keratins on chromosome 12. The other
listed options do not have active keratin functions.

25) Tyrosinase is the enzyme that catalyzes the conversion fo tyrosine to DOPA and DOPA to
DOPAquinone. The enzyme contains which of the following ions?




                                                  7
A. Iron
B. Magnesium
C. CopperCorrect Choice
D. Selenium
E. Zinc
Tyrosinase is a copper containing enzyme that is responsible for the conversion fo tyrosine to
DOPA and DOPA to DOPAquinone.

26) Choose the correct answer regarding melanin and skin color

A. Facultative skin color is the amount of cutaneous melanin pigment generated according to
cellular genetics
B. In black and brown skin the melanosomes are smaller in diameter and length
C. The number of melanocytes increases with one exposure to UVA/visible light
D. In white skin the melanosomes form groups within the secondary lysosomes Correct Choice
E. Eumelanin produces a yellow chromophore
Melanocytes of dark skin synthesize melanosomes larger than those produced in light skin (thus
(A) is incorrect). The number of melanocytes in the epidermis is the same, regardless of the
person's race or color: it is the number and size of the melanosomes or pigment granules,
continuously synthesized by these melanocytes, that determine differences in skin color (thus (E)
& (B) are incorrect). The size of th melanosome is the principle factor in determining how the
melanosomes will be distributed within the keratinocytes. The larger the melanosomes of dark
skin are individually dispersed within the cytoplasm of keratinocytes: smaller melanosomes of
light skin are packaged in membrane-bound complexes within keratinocytes (thus (C) is correct).
Eumelanin is in dark oval melanosomes found in black hair (thus (D) is incorrect

27) Each of the following is true about melanosomes except:

A. Are transferred to keratinocytes via phagocytosis
B. Tyrosinase activity decreases as melanosomoes mature
C. Most characteristic organelle of the melanocyte
D. Are singly dispersed in the basal layer in white skinCorrect Choice
E. Are larger in size in black skin compared to white skin
Several differences exist that may explain the heterogeneity of skin color. The number of
melanocytes are the same; however, there are several differences in the melanosomes.
Differences in skin color can be attributed to five factors. In racially heavily pigmented skin, (1)
there is greater production of melanosomes in melanocytes, (2) individual melanosomes show a
higher degree of melanization, (3) melanosomes are larger, (4) the melanosomes are dispersed
to a greater degree in the keratinocytes, and (5) there is a slower rate of degradation.

28) Which cytokeratin would you expect to be preferentially expressed in the keratinocytes of a
psoriatic plaque

A. K5
B. K1
C. K12


                                                  8
D. K15
E. K16Correct Choice
K16 and K6 act as markers for hyperproliferative keratinocytes. They are found in skin disease
such as psoriasis, warts, actinic keratoses, and SCC's

29) All of the following are true regarding the formation of hair except:

A. The hair matrix becomes the hair and the inner root sheath
B. The outer root sheath is a downward extension of the epidermis
C. Huxley’s layer contains melaninCorrect Choice
D. Henle’s layer is outside of Huxley’s layer
E. The inner root sheath keratinizes by means of trichohyalin granules
The inner root sheath is composed of three layers, which are the inner root sheath cuticle, Huxley
layer, Henle layer. None of these layers contain melanin. All three layer keratinize by
trichohyaline granules and disintegrate when they reach the isthmus of the hair follicle.

30) The hair follicle is the site of production or conversion of which of the following hormones?

A. adrenocorticotropin hormone (ACTH)
B. prolactin
C. dihydrotestosterone (DHT)
D. alpha-melanocyte stimulating hormone (MSH)
E. All the options are correctCorrect Choice
The hair follicle converts gonadal or aderenal testosterone to dihydrotestosterone locally via 5-
alpha reductase. It also cleaves proopiomelanocortin to ACTH, alpha-MSH, and B-endorphin via
prohormone convertase. It also secretes corticotropin releasing hormone (CRH) as well as
prolactin.

31) Sebaceous glands are located in each of the following locations except:

A. Buccal mucosa
B. Eyelids
C. Nipple
D. Palms Correct Choice
E. Labia minora
Sebaceous glands secrete their contents in a holocrine fashion and are primarily under the
influence of androgens. They secrete triglycerides, phospholipids, esterified cholesterol, waxes
but not free cholesterol. They may be found in association with hair follicles or in some areas of
modified skin such as the nipple/areola, labia minora, prepuce, vermilion border, and eyelids.

32) Which keratins are upregulated in hyperproliferative disease such as psoriasis?

A. Keratins 5 and 14
B. Keratins 2e and 10



                                                  9
C. Keratins 6 and 16Correct Choice
D. Keratins 8 and 18
E. Keratins 1 and 10
Keratins 6 and 16 are upregulated in hyperproliferative keratinocytes, outer root sheath, and oral
epithelium. A defect in these keratins may result in pachyonychia congenita.

33) Total epidermal renewal time (including dividing, movement through the granular layer, and
being shed from the stratum corneum) is approximately

A. 9-25 days
B. 29-35 days
C. 1-4 days
D. 59-75 daysCorrect Choice
E. 5-7 days
A cell takes about 19 days to divide, 26-42 days to move up through the granular layer, and 14
days to shed from the stratum corneum. Therefore the total epidermal renewal time is about
59-75 days

34) Markers for epidermal proliferation seen in psoriasis and actinic keratoses are

A. K1/K9
B. K2/K9
C. K5/K14
D. K6/K16Correct Choice
E. K6/K16
Plectin is found in the basement membrane zone and functions to link intermediate filaments to
the plasma membrane. Mutations in plectin result in epidermolysis bullosa associated with
muscular dystrophy. Plectin is a member of the plakin family. Other members of this family are
envoplakin, periplakin, desmoplakin, and BP230 (BPAg1).

35) Which of the following keratins would most likely be expressed in the nail bed?

A. K6b/17Correct Choice
B. K6a/16
C. K1/9
D. K2e/10
E. K4/13
Keratins 6b & 17 are expressed in the nail bed. K6a/16 is expressed in the outer root sheath and
in hyperproliferative keratinocytes, 1/9 in palmoplantar suprabasalar keratinocytes, 2e/10 in the
upper spinous and granular cell layers, and 4/13 are expressed in mucosal epithelium.

36) What is the major function of urocanic acid?




                                                10
A. primarily a UVA filterCorrect Choice
B. primarily a UVB filter
C. aids in protecting the skin from dermatophytes
D. helps degrade free fatty acids
E. bacteriocidal acid produced by stratum corneum
Urocanic acid a by product of filaggrin degradation and has a peak absorbtion of 345 nm. It
serves as a major UVA filter.

37) Hair grows at

A. 10 mm/day
B. 0.004 mm/day
C. 0.04 mm/day
D. 4 mm/day
E. 0.4 mm/dayCorrect Choice
Hair grows at an average of 0.4 mm/day. The other options are incorrect.

38) Which of the following dermal cells always express CD11c and CD6?

A. Mononuclear phagocytic cellsCorrect Choice
B. Glomus cells
C. Dermal dendrocytes
D. Fibroblasts
E. Mast cells
The mononuclear phagocytic cell includes monocytes, macrophages and dermal dendrocytes. All
phagocytic skin macrophages express CD11c and CD6. Fibroblasts do not produce a CD marker.
Mast cells are derived from bone marrow-residing CD34+ stem cells. They do not produce CD11c
or CD6. Dermal dendrocytes do not express these marker either.

39) Which of the following domains is targeted by the autoantibodies in bullous pemphigoid?

A. NC16A of BP230
B. alpha-6 integrin
C. NC16A of BP180Correct Choice
D. Laminin 5
E. Plectin
The NC16A domain of BP180 is the target of autoantibodies in bullous pemphigoid. It interacts
with alpha-6 integrin extracellularly

40) Which of the following statements is true about eccrine glands?

A. postganglionic sympathetic fibers with acetylcholine as the principal neurotransmitterCorrect



                                                11
Choice
B. postganglionic sympathetic fibers with both norepinephrine and acetylcholine as the principal
neurotransmitters
C. postganglionic parasympathetic fibers with norepinephrine as the principal neurotransmitter
D. postganglionic parasympathetic fibers with acetylcholine as the principal neurotransmitter
E. postganglionic sympathetic fibers with norepinephrine as the principal neurotransmitter
Eccrine glands are innervated by postganglionic sympathetic fibers with acetylcholine as the
principal neurotransmitter. This explains why medications associated with anticholinergic side
effects may be associated with hypohidrosis.

41) Eccrine glands are found in all the following areas of the body except:

A. Labia minoraCorrect Choice
B. Scalp
C. Axillae
D. Cutaneous lip
E. Palms
Eccrine glands are sweat glands enervated by cholinergic sympathetic nerves mediated by
acetylcholine. They absent on modified skin which lacks appendages like the vermillion border,
nail beds, glans penis, inner aspect of the prepuce and the labia minora.

42) Which of the following statements about the direct immunofluorescence pattern in lichen
planus is correct?

A. Deposition of IgG is within cytoid bodies in the superficial dermis
B. Deposition of fibrinogen is within cytoid bodies in the deep dermis
C. The DEJ deposition is granularCorrect Choice
D. There is prominent deposition of IgM within the spinous layer of the epidermis.
E. The DIF is negative in the vast majority of cases
The DEJ deposition is granular. The DIF is positive in the vast majority of cases. Deposition of
IgM and fibrinogen is within cytoid bodies in the superficial dermis. There is no deposition of IgM
within the spinous layer of the epidermis.

43) Which of the following locations has eccrine glands?

A. under nails
B. labia majoraCorrect Choice
C. penis
D. ear canal
E. labia minora
Eccrine glands are enervated by cholinergic sympathetic nerves, mediated by acetycholine. They
are absent on modified skin that lack appendages such as the vermillion border, nail bed, labia
minora, glans penis and ear canal. They are present of the labia majora.


                                                 12
44) of linking proteins?

A. desmocollin
B. plakoglobinCorrect Choice
C. E-cadherin
D. periplakin
E. envoplakin
Plakoglobin is an example of an armadillo protein, which links the cytoskeleton associated linking
proteins (such as plakins in the case of intermediate filaments and alpha-catenin in the case of
actin) to the transmembrane adhesion molecules, termed cadherins. E-cadherin and desmocollin
are examples of cadherins. Periplakin and envoplakin are examples of plakin linking proteins.

45) Hair follicle development in the human embryo begins during

A. Within 2 weeks of fertiization
B. 3rd trimester
C. At the blastocoele stage
D. 2nd trimester
E. 1st trimesterCorrect Choice
Hair follicles begin in the third month of fetal life as a downgrowth of cells from the epidermis (3rd
month=12th week=1st trimester)

46) Which of the following cells are required for wound healing?

A. Lymphocyte
B. Eosinophil
C. MacrophageCorrect Choice
D. Neutrophil
E. Langerhans cell
The macrophage is required for wound healing. The macrophages debride tissue, secrete
collagenase and stimulate expression of FGF, IL-1, TGF-beta, PDGF and TGF-alpha thus
facilitating transition from inflammation to repair.
47) Anagen effluvium

A. All of these options are correctCorrect Choice
B. Stimulus induces the abrupt cessation of mitotic activity in the rapidly dividing hair matrix cells
C. Frequently seen sollowing administration of cancer chemotherapeutic agents
D. Occurs within days to weeks of exposure
E. Is entirely reversible
All of the above statements regarding anagen effluvium are correct. Frequent causes include
antimetabolites, alkylating agents, mitotic inhibitors, thallium, boron.



                                                  13
48) What component is the major barrier in the stratum corneum?

A. Triglycerides
B. Squalene
C. Sebum
D. Collagen
E. CeramideCorrect Choice
The major component of lamellar granules of the keratinocytes is ceramide. These play a major
role in the barrier function of the skin.
49) Meibomian glands are

A. apocrine glands found on the eylelids.
B. eccrine glands localized to the vermillion border of the lips.
C. sebaceous glands found on the areola of the breast.
D. apocrine glands found in the anogenital regions.
E. sebaceous glands found on the eyelids. Correct Choice
Sebaceous glands enlarge at puberty in response to increased levels of androgens. They are
holocrine glands. Meibomian glands are modified sebaceous glands found in the eyelids. Free
sebaceous glands not associated with hairs are found in the nipple and areola and are called
Montgomery’s tubercles. Fordyce’s condition involves free sebaceous glands on the vermillion
border of the lips and on the buccal mucosa. Sebaceous glands are found everywhere on the
skin except the palms and soles.

50) A salt split skin DIF is performed on a biopsy taken adjacent to the skin lesions shown. Where
would you expect staining to be seen?

A. Dermal side
B. Epidermal and Dermal sides equally
C. In the anchoring plaques
D. In the lamina densa
E. Epidermal sideCorrect Choice
The image shown is bullous pemphigoid. On salt split skin DIF exams, deposits are seen on the
epidermal side of the split. If dermal deposits are seen, epidermolysis bullosa acquisita or anti-
epiligrin pemphigoid are potential diagnoses

51) Pick the correctly paired keratin with its structure

A. K3/K12 - esophagus
B. K16/K6 - palms and solesCorrect Choice
C. K5/K14 - suprabasal cells
D. K4/K13 - cornea
E. K1/K10 - basal cells




                                                  14
Keratin 1 and 10 are found in the stratum spinosum (thus (A) is incorrect). K5 and K14 are found
in the basal layer (thus (D) is incorrect). K3 and K12 are found in the suprabasilar cells of the
cornea (thus (B) is incorrect). K4 and K13 are found in the non-cornifying cells of stratified
mucosa (thus (C) is incorrect). K16 and K6 are found in the palms and soles (thus (E) is correct).

52) Defects in what kind of structural protein lead to pyloric atresia associated with junctional
epidermolysis bullosa:

A. Integrin Correct Choice
B. Loricrin
C. Collagen
D. Elastin
E. Plectin
Junctional epidermolysis bullosa with pyloric atresia involves a defect in the b4 subunit of the
a6b4 integrin. The expression of this protein is limited to the basal layer of the epidermis. This
integrin is a transmembrane protein that coordinates a link between the intermediate filaments
(keratins) and the extracellular matrix of the basement membrane. The b4 domain mediates an
interaction with both plectin and BP180; its absence prevents hemidesmosomal assembly.

53) Moving from internally to externally choose the correct description of the hair follicle:

A. Henle’s layer _ Huxley’s layer _ inner root sheath cuticle _ hair shaft cuticle _ cortex _ medulla
B. inner root sheath cuticle _ outer root sheath cuticle _ hair shaft cuticle _ cortex _ Huxley’s layer
_ Henle’s layer
C. inner root sheath cuticle _ Huxley’s layer _ Henle’s layer _ medulla _ cortex _ hair shaft cuticle
D. hair shaft cuticle _ cortex _ medulla _ Henle’s layer _ Huxley’s layer _ inner root sheath cuticle
E. medulla _ cortex _ hair shaft cuticle _ inner root sheath cuticle _ Huxley's layer _ Henle's layer
Correct Choice
(As described in correct choice)

54) Homocystinuria has abnormal crosslinking of collagen because of a mutation in:

A. cystathione synthaseCorrect Choice
B. lysyl oxidase
C. lysyl hydroxylase
D. type I collagen N-peptidase gene
E. Tenascin X
Homocystinuria is caused by a mutation in cystathione synthase. The main skin findings are a
malar flush, livedo reticularis and leg ulcerations. A characteristic eye finding is the downward
displacement of the lens. The other options are involved in abnormalities associated with Ehlers-
Danlos syndrome. Lysyl hydroxylase is deficient in Kyphoscoliosis type of EDS. Tenascin X is
involves in ~3% of Classical type EDS cases. Dermatosparaxis type EDS has recessive
mutations in the type I collagen N-peptidase gene.

55) At what estimated gestational age are all layers of the keratinized epidermis identifiable?




                                                  15
A. 20 weeks
B. 12 weeks
C. 16 weeks
D. 24 weeksCorrect Choice
E. 8 weeks
At 24 weeks, all the layers of the mature epidermis can be identified, and the epidermis is
keratinized

56) Which of the following is NOT characterized by IgG and/or C3 at the basement membrane
zone?

A. Mucosal pemphigoid
B. Pemphigus vulgarisCorrect Choice
C. Bullous pemphigoid
D. Pemphigoid gestationis
E. Epidermolysis bullosa acquisita
All of the listed conditions except pemphigus vulgaris have IgG and/or C3 deposits at the
basement membrane zone. Pemphigus vulgaris is characterized by IgG deposition in the
intercellular space of the epidermis.

57) Keratinocytes have been shown to secrete all of the following cytokines except:

A. IL-1
B. IL-6
C. TNF-alpha
D. IL-8
E. IL-2Correct Choice
Keratinocytes have been shown to secrete all of the above cytokines, except IL-2, IL-4, and IFN-
gamma

58) Which one of the following is responsible for maintaining a barrier to water loss in the stratum
corneum?

A. involucrin
B. transglutaminase
C. loricrin
D. odland bodiesCorrect Choice
E. filaggrin
Odland bodies, also known as lamellar granules, keratinosomes, and membrane-coating
granules, are small organelles that are discharged from granular cells into the intracellular space
of the granular layer of the epidermis. These bodies have two known functions: they mediate
stratum corneum cell cohesion and they form a barrier to water loss. Odland bodies are round to
oval, measure approximately 300 to 500 nm in diameter, and possess a trilaminar membrane and


                                                 16
a laminated interior. They contain neutral sugars linked to lipids and/or proteins, hydrolytic
enzymes, and free sterols.
Filaggrin is a breakdown product of filaggrin precursor, a component of keratohyaline granules,
which aggregates with keratin filaments and acts as a "glue" for keratin filaments. Involucrin is a
cysteine-rich protein synthesized in the cytoplasm of spinous cells. The enzyme,
transglutaminase, cross-links involucrin in the granular layer forming an insoluble cell boundary
that is resistant to denaturing and reducing chemicals. Loricrin, is a highly insoluble sulfur- and
glycine/serine-rich protein, which is the major protein comprising the cornified cell envelope.

(Lever's Histopathology of the Skin. 8th ed 1997 p11)
(Fitzpatrick's Dermatology in General Medicine. 5th ed 1999, p 136-138)

59) Which signaling molecule mediates the transition of hair cycling from telogen to anagen
phase?

A. Fgf5
B. Foxn1
C. 5a-reductase
D. Dihydotestosterone
E. Sonic hedgehogCorrect Choice
Sonic hedgehog (Shh), a signaling molecule secreted by ectodermal cells of the developing hair
follicle, appears to be critical in mediating the transition from telogen to anagen during postnatal
hair cycling

60) Apocrine glands

A. demonstrate decapitation secretion. Correct Choice
B. are fully functional at birth.
C. are thermoregulatory.
D. are diffusely distributed on the body.
E. demonstrate holocrine secretion.
Apocrine glands show decapitation secretion. Like eccrine glands, apocrine glands are composed
of three segments, the intraepidermal duct, the intradermal duct, and the secretory portion. The
duct of the apocrine gland usually leads to a pilosebaceous follicle above the entrance of the
sebaceous duct. Apocrine glands are found in the axillae, anogenital region, external ear canal
(ceruminous glands), in the eyelids (Moll’s glands), and in the breast (mammary glands).
Apocrine glans are functional only at puberty. Their initial secretion is odorless. Hidradenitis
suppuritiva is a disease involving apocrine glands.

61) Which element is necessary for function of matrix metalloproteinases?

A. iron
B. zincCorrect Choice
C. magnesium
D. nitrogen
E. manganese



                                                 17
Matrix metalloproteinases are required for normal tissue architecture and normal turnover of the
extracellular matric. All of them have zinc at the active site and require octahedral binding of
calcium ions to maintain structural integrity.

62) The strength of a scar:

A. None of these options are correct
B. All of these options are correctCorrect Choice
C. Is 20% at 3 weeks
D. Is 5% at 1 week
E. Is 70% at 1 year
A scar has 5% strength at 1 week, 20% at 3 weeks and 70% at 1 year. It will never recover
strength to the level of pre-injury.

63) Which hormone is homologous to alpha-MSH (melanocyte stimulating hormone)?

A. insulin
B. thyroid stimulating hormone
C. prolactin
D. adrenocorticotropic hormoneCorrect Choice
E. human growth factor
Both alpha-MSH and ACTH are cleavage products of proopiomelanocortin (POMC).

64) The microflora of pilosebaceous unit consist of which of the following:

A. Staphylococcus aureus
B. Pseudomonas aeruginosa
C. Pityrosporum ovaleCorrect Choice
D. Escherichia coli
E. Corynebacterium diphtheriae
All the above bacteria and fungi are found within sebaceous glands; the Malasssezia spp. and P.
ovale are found within the acroinfundibulum, S. epidermidis is found within the midinfundibulum,
and Propionibacterium spp. deep within the follicle.

65) Human sebum is distinguished from lipids of internal organs by the presence of:

A. Squalene
B. Wax estersCorrect Choice
C. Glycerides
D. Cholestrol
E. Cholestrol esters
As human sebum exits the sebaceous gland, its major constituents are squalene, cholesterol,
cholesterol esters, triglycerides, and wax esters. With passage through the hair follicle,


                                                18
triyglycerides in the sebum become hydrolyzed by bacterial enzymes, so that by the time the
sebum reaches the skin surface, it contains free fatty acids, mono- and diglycerides in addition to
the original components. Human sebum is distinguished by the presence of wax esters and
squalene. The lipids of human internal organs contain no wax esters and little squalene. The
squalene that is produced in internal organs is quickly converted to lanosterol and then to
cholesterol, so it does not remain in its original form. Human sebaceous glands do not convert
squalene to sterols.


(Fitzpatrick's Dermatology in General Medicine. 5th ed 1999, p [ ] Chapter 10

66) In X-linked ichthyosis, steroid sulfatase is missing from:

A. peroxisomal granules
B. lamellar granulesCorrect Choice
C. basal keratinocytes
D. dermal fibroblasts
E. Odland bodies
Steroid sulfatase is missing from lamellar granules in the stratum corneum in X-linked ichthyosis.
There is an increase in cholesterol sulfate and decreases in cholesterol levels. This may be the
mechanism for the retention hyperkeratosis seen in this condition.

67) Each of the following is true about the basement membrane zone except:

A. Contains laminin 1 and laminin 5
B. Anchoring filaments attach the basal cell membrane to the lamin lucidaCorrect Choice
C. Can be visualized on light microscopy with PAS staining
D. Lamina densa is composed of type IV collagen
E. Anchoring fibrils are composed of type VII collagen
The basement membrane zone is seen on staining with PAS stain. It appears as a homogenous
band approximately 1 micron thick at the dermo-epidermal junction. The hemidesmosomal
complex and basement membrane zone play an integral role in maintaining cellular adhesion.
Anchoring filaments (primarily composed of laminin 5 and BPAG2) attach the basal cell
membrane to the lamina densa NOT lamina lucida.

68) Dystrophic epidermolysis bullosa results from mutations in

A. collagen type III.
B. collagen type II.
C. collagen type I.
D. collagen type VII. Correct Choice
E. collagen type V.
See discussion from question 12

69) Which of the following medications is concentrated in the eccrine glands?




                                                 19
A. all of the answers are correctCorrect Choice
B. cytarabine
C. cephalexin
D. cyclophosphamide
E. ciprofloxacin
The above listed drugs, as well as beta-lactam antibiotics, antifungals such as ketoconazole and
griseofulvin, are known to be secreted into eccrine sweat ducts. This feature may explain the
development of neutrophilic eccrine hidradenitis and eccrine squamous syringometaplasia in the
context of chemotherapy.

70) What is the average duration of the telogen cycle in terminal scalp hair?

A. 9 months
B. 6 months
C. 2-3 weeks
D. 2-6 years
E. 3 monthsCorrect Choice
The average duration of the telogen phase of the hair cycle is 3 months; this feature explains why
telogen effluvium is typically observed 3 months following a traumatic event or serious illness.
The average duration of the anagen phase of the hair cycle is 2-6 years, whereas that of the
catagen cycle is 2-3 weeks.

71) At any one time, the approximate proportion of hair follicles in anagen is

A. 40%
B. 85% Correct Choice
C. 95%
D. 60%
E. 15%
Most hair follicles are in anagen, and thus most hair follicles involve growing hair. The longer a
hair follicle is anagen, the longer the hair can grow in length. Hairs of the scalp grow
approximately 0.4 mm per day, and thus the date of your next hair cut can be accurately
calculated.

72) Desmosine and isodesmosine are typical amino acids found in:

A. Collagen fibers
B. Anchoring plaques
C. Anchoring fibril
D. Elastic fibersCorrect Choice
E. Heparan sulfate
Desmosine and isodesmosine are typical amino acids found in elastic fibers. They crosslink
fibrillin. Anchoring fibrils are composed of collagen VII and collagen fibers and have the most


                                                  20
typical amino acids of proline and hydroxyproline. Heparan sulfate do not typically contain these
amino acids.

73) What is the location of the unbound corticosteroid receptor?

A. mitochondria
B. golgi apparatus
C. nucleus
D. plasma membrane
E. cytoplasmCorrect Choice
Both androgen and corticosteroid receptors localize to the cytoplasm. Estrogen receptors are
found in the nucleus. Progesterone receptors are distributed in both the nucleus and the
cytoplasm

74) The epidermis is comprised of what type of cells?

A. Keratinocytes, Melanocytes, Neutrophils, Langerhan cells
B. Keratinocytes, Endothelial cells, Merkel cells, Goblet cells
C. Keratinocytes, Melanocytes, Merkel cells, Goblet cells
D. Keratinocytes, Endothelial cells, Merkel cells, Langerhan cells
E. Keratinocytes, Melanocytes, Merkel cells, Langerhan cellsCorrect Choice
The adult epidermis is composed of three basic cell types: Keratinocytes, melanocytes, and
Langerhans cells. An additional cell, the Merkel cell, can be found in the basal layer of the palms
and soles, the oral and genital mucosa, the nail bed, and the follicular infundibula.

75) During embryogenesis, periderm cells of the fetus contain which of the following substances?

A. Ceramide
B. Sebum
C. GlycogenCorrect Choice
D. Free fatty acids
E. Porphyrins
In week 7 of embryogenesis the surface ectoderm produces two layers. The external layer is the
periderm which contains glycogen and gives rise to the stratum corneum by week 21. The other
layer is the stratum germinativum.

76) The desmosomal connections of the epidermis are dependent on which of the following ions?

A. CalciumCorrect Choice
B. Iron
C. Zinc
D. Sodium
E. Selenium



                                                 21
The desmosomal connections in the epidermis are calcium dependent. The other options are not
required for these connections.

77) Surgery, Parturition, Fever, Kwashiokor and Hypervitaminosis A are all causes of:

A. Telogen effluviumCorrect Choice
B. Alopecia areata
C. Both telogen and anagen effluvium
D. Anagen effluvium
E. Androgenetic alopecia
All of the listed stressors can induce telogen effluvium. Other causes include traction and some
drug exposures. Usually will only involve up to 50% of scalp hairs and will resolve within 2-3
months

78) Anchoring fibrils are primarily composed of

A. Type II collagen
B. Type IV collagen
C. Type III collagen
D. Type VII collagen Correct Choice
E. Type I collagen
Anchoring fibrils are found in the sublamina densa and are made up of collagen type VII. This
collagen type is mutated in dystrophic epidermolysis bullosa, and targeted in epidermolysis
bullosa acquisita and bullous lupus erythematosus

79) Which eponym describes vestigial lines of pigmentary demarcation?

A. dermatome
B. Langer's lines
C. lines of Blaschko
D. Fuchter linesCorrect Choice
E. Wallace's lines
Fuchter lines are vestigial lines in which the dorsal surface has more melanocytes than ventral
surface. Wallace's lines are the well-demarcated lines around the margin of the foot and hand.

80) Elaunin fibers

A. run parallel in bands within the superficial papillary dermis.
B. run parallel in bands within the reticular dermis. Correct Choice
C. run perpendicular from the dermo-epidermal junction within the superficial papillary dermis.
D. run perpendicular in bands within the deep dermis.
E. run perpendicular in bands within the reticular dermis.




                                                  22
Elaunin fibers are elastic fibers that have less elastin and more fibrillin and run parallel in thin
bands within the reticular dermis. Oxytalin fibers contain no elastin and run perpendicular from
the dermo-epidermal junction within the superficial papillary dermis. Elastic fibers turn over slowly
in the skin, and are damaged by ultraviolet radiation.

81) Sebaceous glands secrete sebum through which of the following secretory mechanisms?

A. Holocrine and Merocrine
B. Merocrine
C. Holocrine and Apocrine
D. HolocrineCorrect Choice
E. Apocrine
Sebaceous glands exhibit holocrine secretion, whereby the sebocytes disintegrate in transit to the
gland center, releasing their sebum contents. Merocrine secretion refers to the formation of
intracellular secretory vesicles that translocate to the apical cell surface for secretion. Apocrine
secretion refers to the process whereby secretory contents are packaged using the apical cell
membrane, and 'pinched off' to achieve secretion.

82) The cutaneous immunofluorescence pattern in patients with Senear-Usher syndrome is:

A. intercellular IgG on guinea pig esophagus
B. intercellular IgG and C3 and linear IgG and C3 along the basement membrane zoneCorrect
Choice
C. linear IgG and C3 along the basement membrane zone
D. linear IgG and C3 along the basement membrane zone and intercellular IgG on guinea pig
esophagus
E. intercellular IgG and C3
Senear-Usher syndrome, or pemphigus erythematosus, is a variant of pemphigus foliaceus
characterized by crusted papules and plaques with a seborrheic distribution (malar region, scalp,
upper chest and back). They demonstrate in-vivo IgG and C3 deposition on keratinocyte cell
membranes and the basement membrane zone, in addition to circulating anti-nuclear antibodies.

83) Sebaceous glands secrete sebum via

A. Exostosis
B. Endocrine mechanism
C. Exocrine mechanism
D. Holocrine mechanismCorrect Choice
E. Mecrocrine mechanism
The sebaceous lobules have basal germinative cells and central sebocytes, which gradually
become more distended with lipid vacuoles until they are shed into the lumen (holocrine
secretion).

84) Which of the following is not a specialized type of sebaceous gland?




                                                 23
A. Fordyce spot
B. Montgomery's tubercle
C. Meibomian gland
D. Gland of Zeis
E. Moll's glandCorrect Choice
There are several types of specialized sebaceous glands that are not associated with a hair
follicle. They include Montgomery's areolar tubercle, Fordyce spots of the lip, Glands of Zeis of
the cutaneous eyelid, and Meibomian glands of the eyelid. Moll's gland of the eyelid are a
modified apocrine gland.

85) Keratohyalin granules contain

A. Envoplakin and Keratin 6
B. Numerous Golgi apparati
C. Desmoplakin
D. Profilaggrin and loricrin Correct Choice
E. Involucrin
Keratohyalin granules are found in the stratum granulosum (the granular layer), and contain the
proteins profilaggrin and loricrin. Profilaggrin is converted to filaggrin during the transformation of
the granular layer to the cornified layer. This is a calcium-dependent process. Loricrin comprises
75% of the cornified envelope’s mass.

86) Which cadherin is responsible for adhesion of Langerhan cells to the epidermis?

A. Desmocollin
B. E-cadherinCorrect Choice
C. P-cadherin
D. N-cadherin
E. Desmoglein
E-cadherin is responsible for the adhesion of Langerhan cells to the epidermis. There are two
major subclasses of cadherins which mediate cell adhesion and play a fundamental role in
normal development, classic (E-,P-,N-cadherin) and desmosomal (desmoglein and desmocollin).
They depend on calcium for their function.

87) Anagen effluvium is best described as

A. a cessation of mitotic activity in rapidly dividing hair matrix cells Correct Choice
B. a scarring alopecia affecting only anagen stage follicles
C. a cessation of mitotic activity in resting hair matrix cells
D. an abrupt transition of telogen to anagen in resting hair matrix cells
E. an abrubt transition from anagen to catagen in rapidly dividing hair matrix cells
Anagen effluvium results from an outside stimulus – most often an antimetabolite,
chemotherapeutic drug – inducing an abrupt cessation of hair matrix cell mitotic activity. This


                                                    24
process occurs within days to weeks of the stimulus, and is reversible with cessation of the drug
therapy.

88) Retinoids upregulate transcription of which types of collagen?

A. 1 and 7Correct Choice
B. 3 and 7
C. 4 and 7
D. 1 and 3
E. 1 and 4
Retinoids upregulate the transcription of collagens one and seven thereby strengthening the
dermis

89) In the epidermis, the cell most responsible for antigen detection and processing is the

A. Melanocyte
B. Merkel Cell
C. Langerhans cell Correct Choice
D. CD4+ T cell
E. Keratinocytes
The Langerhans cell is a bone-derived, antigen-presenting cell found in all layers of the
epidermis, oral mucosa, esophagus, and vagina. Langerhans cells ingest and process antigens,
mature, migrate to a local lymph node, and then present the antigen to a naïve (or resting) T cell,
activating that T cell. The Langerhans cell is central to the pathogenesis of atopic dermatitis,
psoriasis, allergic contact dermatitis, and certain infections, such as Leishmaniasis.

90) The formation of granulation tissue depends on the presence of

A. platelets.
B. collagen type I.
C. collagen type IV.
D. fibronectin. Correct Choice
E. neutrophils.
See discussion of wound healing from questions 29 through 33.

91) People with darker skin show

A. a grouping of melanosomes with a low degree of melanization
B. a lessened production of melanosomes within melanocytes
C. a higher degree of dispersion of melanosomes in keratinocytes Correct Choice
D. smaller, more concentrated melanosomes
E. a more rapid degradation of melanosomes




                                                25
More darkly pigmented races show a greater production of melanosomes in the melanocyte,
melanosomes with a higher degree of melanization, larger melanosomes, a higher degree of
dispersion of melanosomes in the keratinocytes, and a slower rate of melanosome degradation.

92) Which of the following enzymes does not require copper for functioning?

A. tyrosinase
B. cystathione beta-synthase
C. ferrochelataseCorrect Choice
D. lysyl oxidase
E. ATP7a
All of the listed enzymes are copper containing or dependent except ferrochelatase. Lysyl
oxidase facilitates crosslinking of fibrillin in elastic fibers. ATP7a is deficient in Menkes Kinky Hair
Syndrome. Cystathione beta-synthase is defective in homocystinuria. Tyrosinase catalyzes the
first 2 steps, and at least 1 subsequent step, in the conversion of tyrosine to melanin.
Ferrochelatase mutation leads to excess protoporphyrin production and photosensitivity.

93) When do melanocytes begin to synthesize melanin?

A. 2nd month of gestation
B. 4th month of gestation
C. 3rd month of gestationCorrect Choice
D. 6th month of gestation
E. 5th month of gestation
Melanocytes begin to synthesize melanin in the 3rd month of gestation.

94) Red or blonde hair pigmentation primarily results from

A. the reduced activity of DOPA dehydroxylase.
B. the presence of eumelanin.
C. the presence of pheomelanin. Correct Choice
D. the reduced activity of tyrosinase.
E. the absence of melanin.
Hair color is determined by melanocytes. The melanocytic activity of follicular melanocytes is
coupled to anagen – hair is only pigmented when it is growing. Pigment is produced in the matrix
area of follicle, above the follicular papilla. Eumelanin is the pigment of brown/black hairs, and
pheomelanin is the pigment of red/blonde hairs. Intensity of color is proportional to the amount of
pigment. The absence of pigment produces white hair, and markedly reduced pigment produces
gray hair.

95) Sebaceous glands

A. respond to adrenergic neural activity, exclusively.
B. respond to the local release of cytokines from inflammatory cells.




                                                   26
C. respond to chemical stimuli such as hormones. Correct Choice
D. respond to cholinergic neural activity, exclusively.
E. respond to both adrenergic and cholinergic stimuli.
Sebaceous glands are androgen-responsive holocrine glands that enlarge at puberty. Meibomian
glands of the eyelids are modified sebaceous glands. Sebaceous glands are found everywhere
on the skin except the palms and soles. Fordyce’s condition involves free sebaceous glands on
the vermillion border of the lips and on the buccal mucosa. Eccrine glands are thermoregulatory
structures that respond to cholinergic stimulation.

96) Anchoring filaments originate at the hemidesmosomes and insert into the

A. Sub basal dense plate
B. BPAG 180
C. Lamina lucida
D. Lamina densaCorrect Choice
E. Desmosome
Anchoring filaments (smaller than anchoring fibrils) stretch from the plasma membrane through
the subbasal dense plaque and the lamina lucida to the lamina densa

97) Keratinocytes are derived from which of the following:

A. Endoderm
B. Neural Crest
C. Bone marrow precursors
D. EctodermCorrect Choice
E. Mesoderm
As implied by the root 'ecto', a prefix meaning "outer", the keratinocytes of the epidermis are
derived from the ectoderm. The other layers do contribute cell populations that are present in the
skin.




                                                  27

More Related Content

What's hot

Arab board primary exam in dermatology 2012
Arab board primary exam  in dermatology 2012Arab board primary exam  in dermatology 2012
Arab board primary exam in dermatology 2012Derma202
 
Dermatology board review
Dermatology board reviewDermatology board review
Dermatology board reviewAhmed Amer
 
MCQ July with answers - Dr Ameen Alawadhi
MCQ July with answers - Dr Ameen AlawadhiMCQ July with answers - Dr Ameen Alawadhi
MCQ July with answers - Dr Ameen Alawadhiaskadermatologist
 
MCQ August with answers - Dr Ameen Alawadhi
MCQ August with answers - Dr Ameen AlawadhiMCQ August with answers - Dr Ameen Alawadhi
MCQ August with answers - Dr Ameen Alawadhiaskadermatologist
 
Dermatology MCQ and AAFP.pptx
Dermatology MCQ and AAFP.pptxDermatology MCQ and AAFP.pptx
Dermatology MCQ and AAFP.pptxAbdulaziz Bagasi
 
Looking through a Dermoscope
Looking through a DermoscopeLooking through a Dermoscope
Looking through a DermoscopeYogesh Kalyanpad
 
Non Infectious Granulomatous Disorders
Non Infectious Granulomatous DisordersNon Infectious Granulomatous Disorders
Non Infectious Granulomatous DisordersIbrahim Farag
 
Keratinocytes And Keratinization Gammmeeel
Keratinocytes And Keratinization GammmeeelKeratinocytes And Keratinization Gammmeeel
Keratinocytes And Keratinization GammmeeelM.YOUSRY Abdel-Mawla
 
Biologicals in Deramtology (Part 1 )
Biologicals in Deramtology (Part 1 )Biologicals in Deramtology (Part 1 )
Biologicals in Deramtology (Part 1 )Smruti Ramawanshi
 
Epidermopoeisis - development of skin
Epidermopoeisis - development of skin Epidermopoeisis - development of skin
Epidermopoeisis - development of skin Kriti Maheshwari
 
Epidermal kinetics
Epidermal kineticsEpidermal kinetics
Epidermal kineticsRohit Singh
 
Approach to photodermatoses
Approach to photodermatosesApproach to photodermatoses
Approach to photodermatosesDrYusraShabbir
 
Dermatology
DermatologyDermatology
DermatologyDerma202
 

What's hot (20)

Arab board primary exam in dermatology 2012
Arab board primary exam  in dermatology 2012Arab board primary exam  in dermatology 2012
Arab board primary exam in dermatology 2012
 
Dermatology board review
Dermatology board reviewDermatology board review
Dermatology board review
 
MCQ July with answers - Dr Ameen Alawadhi
MCQ July with answers - Dr Ameen AlawadhiMCQ July with answers - Dr Ameen Alawadhi
MCQ July with answers - Dr Ameen Alawadhi
 
MCQ August with answers - Dr Ameen Alawadhi
MCQ August with answers - Dr Ameen AlawadhiMCQ August with answers - Dr Ameen Alawadhi
MCQ August with answers - Dr Ameen Alawadhi
 
Derma.
Derma.Derma.
Derma.
 
Ganyang MCQ Dermatology
Ganyang MCQ DermatologyGanyang MCQ Dermatology
Ganyang MCQ Dermatology
 
Dermatology MCQ and AAFP.pptx
Dermatology MCQ and AAFP.pptxDermatology MCQ and AAFP.pptx
Dermatology MCQ and AAFP.pptx
 
Looking through a Dermoscope
Looking through a DermoscopeLooking through a Dermoscope
Looking through a Dermoscope
 
Non Infectious Granulomatous Disorders
Non Infectious Granulomatous DisordersNon Infectious Granulomatous Disorders
Non Infectious Granulomatous Disorders
 
Keratinocytes And Keratinization Gammmeeel
Keratinocytes And Keratinization GammmeeelKeratinocytes And Keratinization Gammmeeel
Keratinocytes And Keratinization Gammmeeel
 
Dermoscopy an overview
Dermoscopy  an overviewDermoscopy  an overview
Dermoscopy an overview
 
Biologicals in Deramtology (Part 1 )
Biologicals in Deramtology (Part 1 )Biologicals in Deramtology (Part 1 )
Biologicals in Deramtology (Part 1 )
 
Acne presentation
Acne presentationAcne presentation
Acne presentation
 
Epidermopoeisis - development of skin
Epidermopoeisis - development of skin Epidermopoeisis - development of skin
Epidermopoeisis - development of skin
 
Epidermal kinetics
Epidermal kineticsEpidermal kinetics
Epidermal kinetics
 
Approach to photodermatoses
Approach to photodermatosesApproach to photodermatoses
Approach to photodermatoses
 
Nutrophilic dermatosis
Nutrophilic dermatosisNutrophilic dermatosis
Nutrophilic dermatosis
 
Dermatology
DermatologyDermatology
Dermatology
 
Atopic Dermatitis
Atopic DermatitisAtopic Dermatitis
Atopic Dermatitis
 
Interface dermatitis tutorial
Interface dermatitis tutorialInterface dermatitis tutorial
Interface dermatitis tutorial
 

Viewers also liked

dermatology.1 eryth, telan, urt & hshp(dr.faraydwn)
dermatology.1 eryth, telan, urt & hshp(dr.faraydwn)dermatology.1 eryth, telan, urt & hshp(dr.faraydwn)
dermatology.1 eryth, telan, urt & hshp(dr.faraydwn)student
 
Dermatology osce slides
Dermatology osce slidesDermatology osce slides
Dermatology osce slidesHabrol Afzam
 
ETAS_MCQ_11 disorder of hair and nails
ETAS_MCQ_11 disorder of hair and nailsETAS_MCQ_11 disorder of hair and nails
ETAS_MCQ_11 disorder of hair and nailsDerma202
 
ETAS_MCQ_15 dermatologic and cosmetic surgery
ETAS_MCQ_15 dermatologic and cosmetic surgeryETAS_MCQ_15 dermatologic and cosmetic surgery
ETAS_MCQ_15 dermatologic and cosmetic surgeryDerma202
 
Cutaneous Manifestations of GI Malignancies
Cutaneous Manifestations of GI MalignanciesCutaneous Manifestations of GI Malignancies
Cutaneous Manifestations of GI MalignanciesMohammed Ezz El-din
 

Viewers also liked (7)

dermatology.1 eryth, telan, urt & hshp(dr.faraydwn)
dermatology.1 eryth, telan, urt & hshp(dr.faraydwn)dermatology.1 eryth, telan, urt & hshp(dr.faraydwn)
dermatology.1 eryth, telan, urt & hshp(dr.faraydwn)
 
Dermatology osce slides
Dermatology osce slidesDermatology osce slides
Dermatology osce slides
 
ETAS_MCQ_11 disorder of hair and nails
ETAS_MCQ_11 disorder of hair and nailsETAS_MCQ_11 disorder of hair and nails
ETAS_MCQ_11 disorder of hair and nails
 
ETAS_MCQ_15 dermatologic and cosmetic surgery
ETAS_MCQ_15 dermatologic and cosmetic surgeryETAS_MCQ_15 dermatologic and cosmetic surgery
ETAS_MCQ_15 dermatologic and cosmetic surgery
 
Fe
FeFe
Fe
 
Serofuloderma
SerofulodermaSerofuloderma
Serofuloderma
 
Cutaneous Manifestations of GI Malignancies
Cutaneous Manifestations of GI MalignanciesCutaneous Manifestations of GI Malignancies
Cutaneous Manifestations of GI Malignancies
 

Similar to ETAS_MCQ_01 structures of skin

Structure of skin
Structure of skin Structure of skin
Structure of skin docjikisha
 
Cell adhesion molecules
Cell adhesion moleculesCell adhesion molecules
Cell adhesion moleculesNahla Imbarak
 
FUNCTION OF SKIN (2).pptx
FUNCTION OF SKIN (2).pptxFUNCTION OF SKIN (2).pptx
FUNCTION OF SKIN (2).pptxTEJARAM19
 
05 inst guide
05 inst guide05 inst guide
05 inst guideTheSlaps
 
Simple squamous epithelium is structured to best provideA. Diffus.pdf
Simple squamous epithelium is structured to best provideA. Diffus.pdfSimple squamous epithelium is structured to best provideA. Diffus.pdf
Simple squamous epithelium is structured to best provideA. Diffus.pdfarpittradersjdr
 
6. stem cells+regeneration dr ashutosh kumar
6. stem cells+regeneration dr ashutosh kumar6. stem cells+regeneration dr ashutosh kumar
6. stem cells+regeneration dr ashutosh kumarDrAshutosh Kumar
 
Biology of keratinocytes
Biology of keratinocytesBiology of keratinocytes
Biology of keratinocytesHima Farag
 
skin and derivatives.pdf
skin and derivatives.pdfskin and derivatives.pdf
skin and derivatives.pdfHema752685
 
Gingival Keratinization
Gingival KeratinizationGingival Keratinization
Gingival KeratinizationAditiRao48
 
Immune privileged tissues
Immune privileged tissuesImmune privileged tissues
Immune privileged tissuesCellGenuity
 
Skin Tissue - Histology and Embryrology.
Skin Tissue - Histology and Embryrology.Skin Tissue - Histology and Embryrology.
Skin Tissue - Histology and Embryrology.Denizasmaz
 
Defence mechanism of gingiva
Defence mechanism of gingivaDefence mechanism of gingiva
Defence mechanism of gingivaManishaSinha17
 
chapter2basicimmunologypptsdz20102-190704152355.pdf
chapter2basicimmunologypptsdz20102-190704152355.pdfchapter2basicimmunologypptsdz20102-190704152355.pdf
chapter2basicimmunologypptsdz20102-190704152355.pdfNadiirMahamoud
 
8- Which of the following is a true statement- A- Myelin and acetyleho.pdf
8- Which of the following is a true statement- A- Myelin and acetyleho.pdf8- Which of the following is a true statement- A- Myelin and acetyleho.pdf
8- Which of the following is a true statement- A- Myelin and acetyleho.pdfaimscomputers1
 
Cells of immune system
Cells of immune systemCells of immune system
Cells of immune systemSrimathiDS
 

Similar to ETAS_MCQ_01 structures of skin (20)

Structure of skin
Structure of skin Structure of skin
Structure of skin
 
Cell adhesion molecules
Cell adhesion moleculesCell adhesion molecules
Cell adhesion molecules
 
FUNCTION OF SKIN (2).pptx
FUNCTION OF SKIN (2).pptxFUNCTION OF SKIN (2).pptx
FUNCTION OF SKIN (2).pptx
 
Integumentary System
Integumentary SystemIntegumentary System
Integumentary System
 
Integumentary System
Integumentary SystemIntegumentary System
Integumentary System
 
Cells in dermis
Cells in dermis   Cells in dermis
Cells in dermis
 
05 inst guide
05 inst guide05 inst guide
05 inst guide
 
Simple squamous epithelium is structured to best provideA. Diffus.pdf
Simple squamous epithelium is structured to best provideA. Diffus.pdfSimple squamous epithelium is structured to best provideA. Diffus.pdf
Simple squamous epithelium is structured to best provideA. Diffus.pdf
 
6. stem cells+regeneration dr ashutosh kumar
6. stem cells+regeneration dr ashutosh kumar6. stem cells+regeneration dr ashutosh kumar
6. stem cells+regeneration dr ashutosh kumar
 
Biology of keratinocytes
Biology of keratinocytesBiology of keratinocytes
Biology of keratinocytes
 
skin and derivatives.pdf
skin and derivatives.pdfskin and derivatives.pdf
skin and derivatives.pdf
 
Gingival Keratinization
Gingival KeratinizationGingival Keratinization
Gingival Keratinization
 
Immune privileged tissues
Immune privileged tissuesImmune privileged tissues
Immune privileged tissues
 
Skin Tissue - Histology and Embryrology.
Skin Tissue - Histology and Embryrology.Skin Tissue - Histology and Embryrology.
Skin Tissue - Histology and Embryrology.
 
Defence mechanism of gingiva
Defence mechanism of gingivaDefence mechanism of gingiva
Defence mechanism of gingiva
 
Skin
SkinSkin
Skin
 
chapter2basicimmunologypptsdz20102-190704152355.pdf
chapter2basicimmunologypptsdz20102-190704152355.pdfchapter2basicimmunologypptsdz20102-190704152355.pdf
chapter2basicimmunologypptsdz20102-190704152355.pdf
 
Chapter 2 Basic Immunology ( Cells & Organs)
Chapter 2 Basic Immunology ( Cells & Organs)  Chapter 2 Basic Immunology ( Cells & Organs)
Chapter 2 Basic Immunology ( Cells & Organs)
 
8- Which of the following is a true statement- A- Myelin and acetyleho.pdf
8- Which of the following is a true statement- A- Myelin and acetyleho.pdf8- Which of the following is a true statement- A- Myelin and acetyleho.pdf
8- Which of the following is a true statement- A- Myelin and acetyleho.pdf
 
Cells of immune system
Cells of immune systemCells of immune system
Cells of immune system
 

More from Derma202

Phototherapy treatment protocol
Phototherapy treatment protocolPhototherapy treatment protocol
Phototherapy treatment protocolDerma202
 
Histopathplogical photos
Histopathplogical photosHistopathplogical photos
Histopathplogical photosDerma202
 
Slide study from ETAS
Slide  study from ETASSlide  study from ETAS
Slide study from ETASDerma202
 
ETAS_MCQ_14 plants and creatures of dermatologic significance
ETAS_MCQ_14 plants and creatures of dermatologic significanceETAS_MCQ_14 plants and creatures of dermatologic significance
ETAS_MCQ_14 plants and creatures of dermatologic significanceDerma202
 
ETAS_MCQ_13 photobiology and photosensitivity disorders
ETAS_MCQ_13 photobiology and photosensitivity disordersETAS_MCQ_13 photobiology and photosensitivity disorders
ETAS_MCQ_13 photobiology and photosensitivity disordersDerma202
 
ETAS_MCQ_03 b genodermatoses
ETAS_MCQ_03 b genodermatosesETAS_MCQ_03 b genodermatoses
ETAS_MCQ_03 b genodermatosesDerma202
 
Derm handbook for medical students and junior doctors 2010
Derm handbook for medical students and junior doctors 2010Derm handbook for medical students and junior doctors 2010
Derm handbook for medical students and junior doctors 2010Derma202
 

More from Derma202 (7)

Phototherapy treatment protocol
Phototherapy treatment protocolPhototherapy treatment protocol
Phototherapy treatment protocol
 
Histopathplogical photos
Histopathplogical photosHistopathplogical photos
Histopathplogical photos
 
Slide study from ETAS
Slide  study from ETASSlide  study from ETAS
Slide study from ETAS
 
ETAS_MCQ_14 plants and creatures of dermatologic significance
ETAS_MCQ_14 plants and creatures of dermatologic significanceETAS_MCQ_14 plants and creatures of dermatologic significance
ETAS_MCQ_14 plants and creatures of dermatologic significance
 
ETAS_MCQ_13 photobiology and photosensitivity disorders
ETAS_MCQ_13 photobiology and photosensitivity disordersETAS_MCQ_13 photobiology and photosensitivity disorders
ETAS_MCQ_13 photobiology and photosensitivity disorders
 
ETAS_MCQ_03 b genodermatoses
ETAS_MCQ_03 b genodermatosesETAS_MCQ_03 b genodermatoses
ETAS_MCQ_03 b genodermatoses
 
Derm handbook for medical students and junior doctors 2010
Derm handbook for medical students and junior doctors 2010Derm handbook for medical students and junior doctors 2010
Derm handbook for medical students and junior doctors 2010
 

ETAS_MCQ_01 structures of skin

  • 1. Structures of skin 1) Dermal dendrocytes A. are the primary cell found in a glomangioma. B. actively synthesize and release IgG. C. are responsible for immediate-type hypersensitivity reactions. D. present antigen to naïve T cells in the lymph node. Correct Choice E. are the primary cell found in an angiosarcoma. A dermal dendrocyte is a mononuclear phagocytic cell that is a type of antigen presenting cell. It is derived from the bone marrow and found both in the papillary and upper reticular dermis. This cell is highly phagocytic and synonymous with the melanophage that has ingested pigment. Dermal dendrocytes are likely very important to the afferent limb of the immune response. 2) What is the major component of the anchoring fibril? A. Type III Collagen B. Type I Collagen C. Type VII CollagenCorrect Choice D. Type IV Collagen E. Laminin 5 Anchoring fibrils are made of Type VII collegen. Anchoring plaques are made of Type IV collagen, and interact with a network of Type I And Type III collagen fibers in the dermis 3) What is the most abundant collagen found on fetal skin? A. Type III CollagenCorrect Choice B. Type II Collagen C. Type IV Collagen D. Type VII Collagen E. Type I Collagen Type III collagen is found in the fetal skin. It is also present in the gastrointestinal tract, blood vessels, and the basement membrane. A defect in this collagen results in the Ehlers-Danlos, vascular type. 4) Mutations in which of the following proteins results in epidermolysis bullosa simplex associated with muscular dystrophy A. Laminin 5 B. Uncein C. collagen type IV D. Plectin Correct Choice E. a6b4 integrin 1
  • 2. Anchoring filaments exist within the lamina lucida. They are primarily comprised of laminin 5 and BP180. Laminin 5 is a cross-shaped assembly of 3 classes of polypeptides, a, b, g. The anchoring filaments function as a structural network to which other proteins attach, and they function as signaling molecules that transmit morphogenetic information to transmembrane proteins of the basal cell layer (such as the integrins). Laminin 5 is also called epiligrin and binds to the a6b4 integrin at the hemidesmosome. 5) The finding on DIF that reflects binding of the Ro and La antigens in subacute cutaneous lupus erythematosus is: A. Immune deposits along the DE junction B. Granular deposits along the basement membrane C. Cytoid bodies D. Granular fluorescence throughout the cytoplasm and nucleus of basal keratinocytesCorrect Choice E. A "chicken-wire" pattern within the epidermis Granular fluorescence throughout the cytoplasm and nucleus of basal keratinocytes reflect the binding of Ro and La antigens and is unique to SCLE. Cytoid bodies and Immune deposits along dermal-epidermal junction are seen in both DLE and SCLE. Granular deposits along the basement membrane are seen in dermatitis herpetiformis and a chicken-wire pattern of staining is seen in pemphigus vulgaris. 6) Which of the following markers are specific and reliable for Merkel cells? A. CD34 B. CK20Correct Choice C. CD20 D. HMB-45 E. CD3 Cytokeratin (CK) 20 is a reliable marker for Merkel cells. CD20 is a marker for B-cells and CD3 is a T-cell marker. HMB-45 is used in staining for immature melanosomes and is reactive in melanoma. CD34 is used to stain dermatofibrosarcoma protuberans. 7) Apocrine glands are found in all of the following areas of the body except: A. Axillae B. Perineum C. Breasts D. Eyelid E. Palms Correct Choice Apocrine glands operate by decapitation secretion and are activated by epinephrine and norepinephrine. They are located in a few distinct areas of the body, which include axillae, anogenital region, Moll’s glands of the eyelids, mammary glands of the breast and the ceruminous glands of the external auditory canal. 8) Which mechanoreceptor found in hair bearing areas sense deep touch and vibration? 2
  • 3. A. Krase end-bulb B. Vater-Pacini corpuscleCorrect Choice C. Merkel cell D. Meissner corpuscle E. Free nerve ending Vater-Pacini corpuscles sense deep touch and vibration. Merkel cells are slow adapting type I mechanoreceptors found among basal keratinocytes. Meissner corpuscles are found in the dermal papilla, especially in the palms and soles. Krase end-bulbs are mucocutaneous end- organs found on the glans penis, prepuce, clitoris, labia minora, and vermillion border of the lip 9) Glomus cells are A. monocytes. B. vascular smooth muscle cells. Correct Choice C. modified fibroblasts. D. neuronal cells. E. modified skeletal muscle cells. Glomus cells are derived from Suquet-Hoyer canals. They are vascular smooth muscle cells that allow the rapid shunting of blood from the arterioral to venular system, bypassing the capillaries. This process occurs primarily on the palms and soles. 10) Which protein is the largest component of the cornified cell envelope? A. Keratin B. Profilaggrin C. Involucrin D. LoricrinCorrect Choice E. Ceramide The cornified cell envelope is a durable, protein-lipid polymer that eventually acts as a mechanical and chemical barrier on the exterior of cornified cells. In the upper spinous layer, keratohyaline granules release profilagrin and loricrin. Profilaggrin is cleaved to filaggrin and subsequently aggregates keratin filaments. Loricrin is the major protein component of the cornified cell envelope and is bound to the cell membrane by transglutaminases (in addition to other structural proteins like involucrin, keratins, elafin, cystatin A and desmosomal peptides). This forms the highly insoluble proteinaceous component of the cornified cell envelope. 11) Embryologically, epidermal stratification occurs at approximately what estimated gestational age? A. 20 weeks B. 16 weeks C. 8 weeksCorrect Choice D. 12 weeks 3
  • 4. E. 4 weeks Epidermal stratification begins at about 8 weeks estimated gestational age and is completed by the 2nd trimester. Epideral stratification occurs when an 'intermediate layer' develops between the epidermal basal cells and the overlying layer of periderm cells. The periderm is an embryonic structure that covers the primitive epidermis until keratinization begins, at which point the periderm sloughs off and contributes to the protective covering of the newborn, the vernix caseosa. This intermediate layer is highly proliferative, such that by 24-25 weeks estimated gestaional age, the epidermis consists of 4 or 5 layers, in addition to the degenerating periderm 12) A specific marker of Merkel cells is A. Envoplakin B. Cytokeratin 10 C. Loricrin D. Cytokeratin 20 Correct Choice E. Cytokeratin 15 Cytokeratin 20 is a specific marker for the Merkel cell. Merkel cells are mechanoreceptors located at body sites requiring high tactile sensitivity. Keratinocyte deformation results in a secretion of chemokines by Merkel cells, which make synaptic connection with neurons. 13) On electron microscopy, which cell demonstrates cytoplasmic projections and secretory granules? A. macrophage B. Langerhans cell C. keratinocyte D. mast cellCorrect Choice E. melanocyte Electron microscopy of mast cells demonstrates large long villi at the periphery and round or oval secretory granules. Langerhans cells show a folded nucleus and rarely phagocytized melanosomes on electron microscopy. Their characteristic feature is the presence of Birbeck granules whose disk shape with one or two vesicles at either end represents a tennis raquet. Examination of melanocytes shows an absence of tonofilaments or desmosomes with characteristic melanosomes in various stages of formation. Macrophages often contain phagocytized material within phagosomes. (Lever's Histopathology of the Skin 8th ed 1997 p.32.) 14) Upon presentation of an antigen in the skin surface, a hapten forms. The first cell to take up the hapten is A. Mast cell B. T cellsCorrect Choice C. B cells D. keratinocytes 4
  • 5. E. Langerhans cells Most of the contact allergens are low-molecular weight chemicals, which after penetrating into the skin, have to couple with host proteins to be able to act as full antigens. These are called haptens. Upon epicutaneous application to a naive host, Langerhans cells take up the hapten, process it and migrate towards the regional lymph nodes, where the anitgen is presented to the naive T cells 15) Elastic fibers contain the specific amino acids A. Alanine and phenylalanine B. Glycine and proline C. Desmosine and isodesmosine Correct Choice D. Leucine and isoleucine E. Lysine and proline Desmosine and isodesmosine are the typical amino acids of elastic fibers. Elastic fibers are comprised of elastin that is wrapped by fibrillin microfibrils. Elastic fibers form a complex meshwork extending from the lamina densa of the dermo-epidermo junction through the dermis. Elastic fibers return the skin to a normal shape after being stretched. 16) Apocrine chromhidrosis results in from which of the following contents of apocrine sweat? A. squalene B. cholesterol C. ammonia D. lipofuschinCorrect Choice E. fatty acids Chromhidrosis refers to the secretion of pigmented sweat, most commonly yellow, green or black. It reflects the rich lipofuschin content of apocrine sweat. Extrinsic apocrine chromhidrosis results from staining of sweat and garments by chromogenic bacteria, such as Corynebacterium spp. 17) The major component of the anchoring filaments is A. Uncein B. Plectin C. Laminin 5 Correct Choice D. a6b4 integrin E. Collagen type IV Anchoring filaments exist within the lamina lucida. They are primarily comprised of laminin 5 and BP180. Laminin 5 is a cross-shaped assembly of 3 classes of polypeptides, a, b, g. The anchoring filaments function as a structural network to which other proteins attach, and they function as signaling molecules that transmit morphogenetic information to transmembrane proteins of the basal cell layer (such as the integrins). Laminin 5 is also called epiligrin and binds to the a6b4 integrin at the hemidesmosome. 18) Which of the following make up the major protein of the cornified cell envelope? 5
  • 6. A. Filaggrin B. LoricrinCorrect Choice C. Involucrin D. Laminin V E. Envoplakin Loricrin is the major protein component of the cornified cell envelope (CE). Involucrin is cross- linked by transglutaminae in the granular layer to form an insoluble cell boundary. Envoplakin may link the CE to desmosomes and to kertin filaments. Filaggrin is thought to promote aggregation and disulfide bonding of keratin filaments in CE. It is degraded into urocanic acid and pyrrolidone carboxylic acid. Both of which hydrate the stratum corneum and block UV radiation. Laminin V is found in the basement membrane and is not involved in the formation of the cornified cell envelope. 19) Tissue contraction begins A. after the 3rd month of wound healing. B. after the first month of wound healing. C. after the 9th month of wound healing. D. at 3rd day of wound healing. E. during the 2nd week of wound healing. Correct Choice See discussion of wound healing from questions 29 through 33. 20) Telogen effluvium A. results from sudden illness or surgery. Correct Choice B. involves a diffuse alopecia affecting more than 50% of the scalp. C. is an inflammatory alopecia devoid of scarring. D. often results from antimetabolites used during cancer chemotherapy. E. is a patchy alopecia affecting less than 50% of the scalp. Telogen effluvium is an excessive loss of club hairs from the normal resting follicles of the scalp. The follicle is pushed from anagen to catagen to telogen. There is no inflammatory process involved. Causes of telogen effluvium are illness, surgery, parturition, fever, drugs, traction, starvation, and hypervitaminosis A. Usually, the hair loss only involves less than 50% of the scalp. There is no specific therapy, and most cases are self-resolving within months. 21) Cutaneous warts in renal transplant recipients show increased expression of which keratin? A. K2 B. K16 C. K5 D. K9 E. K13Correct Choice 6
  • 7. Keratins are divided into acidic (type I) and basic (type II) subtypes. Keratin 13 is a type I keratin which is expressed suprabasally in adult epithelia and is associated with terminal differentiation. Renal transplant recipients have an increased risk of developing skin cancers and multiple warts. Factors contributing to this increased risk include human papillomavirus infection, immunosuppressive therapy, and exposure to ultraviolet radiation. Increased expression of K13 has been demonstrated in high-risk papillomas in this patient population. (Blokx W. Arch Dermatol 2002; 138:61-65 22) Which of the following glands is not under neural control? A. Eccrine glands B. Apocrine glands C. Salivary glands D. Sebaceous glandsCorrect Choice E. Ceruminous glands Sebum is secreted continuously on the skin under hormonal glands. It contains squalene, cholesterol, cholesterol esters, wax, and triglycerides 23) The major protein component of the cornified envelope is A. Plectin B. Envoplakin C. Loricrin Correct Choice D. Transglutaminase E. Desmoplakin Loricrin is the major component of the cornified envelope (CE). The proteins of the CE are synthesized in the spinous and granular layers. The CE is primarily a protein/lipid polymer formed within the differentiating layer of keratinocytes. The CE eventually exists outside of the cornified cells after the granular cell undergoes a programmed destruction (apoptosis). Self-destructing granular cells are called transition cells. 24) Acid keratins (K10-20) are expressed on which of the following chromosomes? A. 12 B. 17Correct Choice C. 18 D. 19 E. 10 Acid keratins are expressed on chromosome 17 and basic keratins on chromosome 12. The other listed options do not have active keratin functions. 25) Tyrosinase is the enzyme that catalyzes the conversion fo tyrosine to DOPA and DOPA to DOPAquinone. The enzyme contains which of the following ions? 7
  • 8. A. Iron B. Magnesium C. CopperCorrect Choice D. Selenium E. Zinc Tyrosinase is a copper containing enzyme that is responsible for the conversion fo tyrosine to DOPA and DOPA to DOPAquinone. 26) Choose the correct answer regarding melanin and skin color A. Facultative skin color is the amount of cutaneous melanin pigment generated according to cellular genetics B. In black and brown skin the melanosomes are smaller in diameter and length C. The number of melanocytes increases with one exposure to UVA/visible light D. In white skin the melanosomes form groups within the secondary lysosomes Correct Choice E. Eumelanin produces a yellow chromophore Melanocytes of dark skin synthesize melanosomes larger than those produced in light skin (thus (A) is incorrect). The number of melanocytes in the epidermis is the same, regardless of the person's race or color: it is the number and size of the melanosomes or pigment granules, continuously synthesized by these melanocytes, that determine differences in skin color (thus (E) & (B) are incorrect). The size of th melanosome is the principle factor in determining how the melanosomes will be distributed within the keratinocytes. The larger the melanosomes of dark skin are individually dispersed within the cytoplasm of keratinocytes: smaller melanosomes of light skin are packaged in membrane-bound complexes within keratinocytes (thus (C) is correct). Eumelanin is in dark oval melanosomes found in black hair (thus (D) is incorrect 27) Each of the following is true about melanosomes except: A. Are transferred to keratinocytes via phagocytosis B. Tyrosinase activity decreases as melanosomoes mature C. Most characteristic organelle of the melanocyte D. Are singly dispersed in the basal layer in white skinCorrect Choice E. Are larger in size in black skin compared to white skin Several differences exist that may explain the heterogeneity of skin color. The number of melanocytes are the same; however, there are several differences in the melanosomes. Differences in skin color can be attributed to five factors. In racially heavily pigmented skin, (1) there is greater production of melanosomes in melanocytes, (2) individual melanosomes show a higher degree of melanization, (3) melanosomes are larger, (4) the melanosomes are dispersed to a greater degree in the keratinocytes, and (5) there is a slower rate of degradation. 28) Which cytokeratin would you expect to be preferentially expressed in the keratinocytes of a psoriatic plaque A. K5 B. K1 C. K12 8
  • 9. D. K15 E. K16Correct Choice K16 and K6 act as markers for hyperproliferative keratinocytes. They are found in skin disease such as psoriasis, warts, actinic keratoses, and SCC's 29) All of the following are true regarding the formation of hair except: A. The hair matrix becomes the hair and the inner root sheath B. The outer root sheath is a downward extension of the epidermis C. Huxley’s layer contains melaninCorrect Choice D. Henle’s layer is outside of Huxley’s layer E. The inner root sheath keratinizes by means of trichohyalin granules The inner root sheath is composed of three layers, which are the inner root sheath cuticle, Huxley layer, Henle layer. None of these layers contain melanin. All three layer keratinize by trichohyaline granules and disintegrate when they reach the isthmus of the hair follicle. 30) The hair follicle is the site of production or conversion of which of the following hormones? A. adrenocorticotropin hormone (ACTH) B. prolactin C. dihydrotestosterone (DHT) D. alpha-melanocyte stimulating hormone (MSH) E. All the options are correctCorrect Choice The hair follicle converts gonadal or aderenal testosterone to dihydrotestosterone locally via 5- alpha reductase. It also cleaves proopiomelanocortin to ACTH, alpha-MSH, and B-endorphin via prohormone convertase. It also secretes corticotropin releasing hormone (CRH) as well as prolactin. 31) Sebaceous glands are located in each of the following locations except: A. Buccal mucosa B. Eyelids C. Nipple D. Palms Correct Choice E. Labia minora Sebaceous glands secrete their contents in a holocrine fashion and are primarily under the influence of androgens. They secrete triglycerides, phospholipids, esterified cholesterol, waxes but not free cholesterol. They may be found in association with hair follicles or in some areas of modified skin such as the nipple/areola, labia minora, prepuce, vermilion border, and eyelids. 32) Which keratins are upregulated in hyperproliferative disease such as psoriasis? A. Keratins 5 and 14 B. Keratins 2e and 10 9
  • 10. C. Keratins 6 and 16Correct Choice D. Keratins 8 and 18 E. Keratins 1 and 10 Keratins 6 and 16 are upregulated in hyperproliferative keratinocytes, outer root sheath, and oral epithelium. A defect in these keratins may result in pachyonychia congenita. 33) Total epidermal renewal time (including dividing, movement through the granular layer, and being shed from the stratum corneum) is approximately A. 9-25 days B. 29-35 days C. 1-4 days D. 59-75 daysCorrect Choice E. 5-7 days A cell takes about 19 days to divide, 26-42 days to move up through the granular layer, and 14 days to shed from the stratum corneum. Therefore the total epidermal renewal time is about 59-75 days 34) Markers for epidermal proliferation seen in psoriasis and actinic keratoses are A. K1/K9 B. K2/K9 C. K5/K14 D. K6/K16Correct Choice E. K6/K16 Plectin is found in the basement membrane zone and functions to link intermediate filaments to the plasma membrane. Mutations in plectin result in epidermolysis bullosa associated with muscular dystrophy. Plectin is a member of the plakin family. Other members of this family are envoplakin, periplakin, desmoplakin, and BP230 (BPAg1). 35) Which of the following keratins would most likely be expressed in the nail bed? A. K6b/17Correct Choice B. K6a/16 C. K1/9 D. K2e/10 E. K4/13 Keratins 6b & 17 are expressed in the nail bed. K6a/16 is expressed in the outer root sheath and in hyperproliferative keratinocytes, 1/9 in palmoplantar suprabasalar keratinocytes, 2e/10 in the upper spinous and granular cell layers, and 4/13 are expressed in mucosal epithelium. 36) What is the major function of urocanic acid? 10
  • 11. A. primarily a UVA filterCorrect Choice B. primarily a UVB filter C. aids in protecting the skin from dermatophytes D. helps degrade free fatty acids E. bacteriocidal acid produced by stratum corneum Urocanic acid a by product of filaggrin degradation and has a peak absorbtion of 345 nm. It serves as a major UVA filter. 37) Hair grows at A. 10 mm/day B. 0.004 mm/day C. 0.04 mm/day D. 4 mm/day E. 0.4 mm/dayCorrect Choice Hair grows at an average of 0.4 mm/day. The other options are incorrect. 38) Which of the following dermal cells always express CD11c and CD6? A. Mononuclear phagocytic cellsCorrect Choice B. Glomus cells C. Dermal dendrocytes D. Fibroblasts E. Mast cells The mononuclear phagocytic cell includes monocytes, macrophages and dermal dendrocytes. All phagocytic skin macrophages express CD11c and CD6. Fibroblasts do not produce a CD marker. Mast cells are derived from bone marrow-residing CD34+ stem cells. They do not produce CD11c or CD6. Dermal dendrocytes do not express these marker either. 39) Which of the following domains is targeted by the autoantibodies in bullous pemphigoid? A. NC16A of BP230 B. alpha-6 integrin C. NC16A of BP180Correct Choice D. Laminin 5 E. Plectin The NC16A domain of BP180 is the target of autoantibodies in bullous pemphigoid. It interacts with alpha-6 integrin extracellularly 40) Which of the following statements is true about eccrine glands? A. postganglionic sympathetic fibers with acetylcholine as the principal neurotransmitterCorrect 11
  • 12. Choice B. postganglionic sympathetic fibers with both norepinephrine and acetylcholine as the principal neurotransmitters C. postganglionic parasympathetic fibers with norepinephrine as the principal neurotransmitter D. postganglionic parasympathetic fibers with acetylcholine as the principal neurotransmitter E. postganglionic sympathetic fibers with norepinephrine as the principal neurotransmitter Eccrine glands are innervated by postganglionic sympathetic fibers with acetylcholine as the principal neurotransmitter. This explains why medications associated with anticholinergic side effects may be associated with hypohidrosis. 41) Eccrine glands are found in all the following areas of the body except: A. Labia minoraCorrect Choice B. Scalp C. Axillae D. Cutaneous lip E. Palms Eccrine glands are sweat glands enervated by cholinergic sympathetic nerves mediated by acetylcholine. They absent on modified skin which lacks appendages like the vermillion border, nail beds, glans penis, inner aspect of the prepuce and the labia minora. 42) Which of the following statements about the direct immunofluorescence pattern in lichen planus is correct? A. Deposition of IgG is within cytoid bodies in the superficial dermis B. Deposition of fibrinogen is within cytoid bodies in the deep dermis C. The DEJ deposition is granularCorrect Choice D. There is prominent deposition of IgM within the spinous layer of the epidermis. E. The DIF is negative in the vast majority of cases The DEJ deposition is granular. The DIF is positive in the vast majority of cases. Deposition of IgM and fibrinogen is within cytoid bodies in the superficial dermis. There is no deposition of IgM within the spinous layer of the epidermis. 43) Which of the following locations has eccrine glands? A. under nails B. labia majoraCorrect Choice C. penis D. ear canal E. labia minora Eccrine glands are enervated by cholinergic sympathetic nerves, mediated by acetycholine. They are absent on modified skin that lack appendages such as the vermillion border, nail bed, labia minora, glans penis and ear canal. They are present of the labia majora. 12
  • 13. 44) of linking proteins? A. desmocollin B. plakoglobinCorrect Choice C. E-cadherin D. periplakin E. envoplakin Plakoglobin is an example of an armadillo protein, which links the cytoskeleton associated linking proteins (such as plakins in the case of intermediate filaments and alpha-catenin in the case of actin) to the transmembrane adhesion molecules, termed cadherins. E-cadherin and desmocollin are examples of cadherins. Periplakin and envoplakin are examples of plakin linking proteins. 45) Hair follicle development in the human embryo begins during A. Within 2 weeks of fertiization B. 3rd trimester C. At the blastocoele stage D. 2nd trimester E. 1st trimesterCorrect Choice Hair follicles begin in the third month of fetal life as a downgrowth of cells from the epidermis (3rd month=12th week=1st trimester) 46) Which of the following cells are required for wound healing? A. Lymphocyte B. Eosinophil C. MacrophageCorrect Choice D. Neutrophil E. Langerhans cell The macrophage is required for wound healing. The macrophages debride tissue, secrete collagenase and stimulate expression of FGF, IL-1, TGF-beta, PDGF and TGF-alpha thus facilitating transition from inflammation to repair. 47) Anagen effluvium A. All of these options are correctCorrect Choice B. Stimulus induces the abrupt cessation of mitotic activity in the rapidly dividing hair matrix cells C. Frequently seen sollowing administration of cancer chemotherapeutic agents D. Occurs within days to weeks of exposure E. Is entirely reversible All of the above statements regarding anagen effluvium are correct. Frequent causes include antimetabolites, alkylating agents, mitotic inhibitors, thallium, boron. 13
  • 14. 48) What component is the major barrier in the stratum corneum? A. Triglycerides B. Squalene C. Sebum D. Collagen E. CeramideCorrect Choice The major component of lamellar granules of the keratinocytes is ceramide. These play a major role in the barrier function of the skin. 49) Meibomian glands are A. apocrine glands found on the eylelids. B. eccrine glands localized to the vermillion border of the lips. C. sebaceous glands found on the areola of the breast. D. apocrine glands found in the anogenital regions. E. sebaceous glands found on the eyelids. Correct Choice Sebaceous glands enlarge at puberty in response to increased levels of androgens. They are holocrine glands. Meibomian glands are modified sebaceous glands found in the eyelids. Free sebaceous glands not associated with hairs are found in the nipple and areola and are called Montgomery’s tubercles. Fordyce’s condition involves free sebaceous glands on the vermillion border of the lips and on the buccal mucosa. Sebaceous glands are found everywhere on the skin except the palms and soles. 50) A salt split skin DIF is performed on a biopsy taken adjacent to the skin lesions shown. Where would you expect staining to be seen? A. Dermal side B. Epidermal and Dermal sides equally C. In the anchoring plaques D. In the lamina densa E. Epidermal sideCorrect Choice The image shown is bullous pemphigoid. On salt split skin DIF exams, deposits are seen on the epidermal side of the split. If dermal deposits are seen, epidermolysis bullosa acquisita or anti- epiligrin pemphigoid are potential diagnoses 51) Pick the correctly paired keratin with its structure A. K3/K12 - esophagus B. K16/K6 - palms and solesCorrect Choice C. K5/K14 - suprabasal cells D. K4/K13 - cornea E. K1/K10 - basal cells 14
  • 15. Keratin 1 and 10 are found in the stratum spinosum (thus (A) is incorrect). K5 and K14 are found in the basal layer (thus (D) is incorrect). K3 and K12 are found in the suprabasilar cells of the cornea (thus (B) is incorrect). K4 and K13 are found in the non-cornifying cells of stratified mucosa (thus (C) is incorrect). K16 and K6 are found in the palms and soles (thus (E) is correct). 52) Defects in what kind of structural protein lead to pyloric atresia associated with junctional epidermolysis bullosa: A. Integrin Correct Choice B. Loricrin C. Collagen D. Elastin E. Plectin Junctional epidermolysis bullosa with pyloric atresia involves a defect in the b4 subunit of the a6b4 integrin. The expression of this protein is limited to the basal layer of the epidermis. This integrin is a transmembrane protein that coordinates a link between the intermediate filaments (keratins) and the extracellular matrix of the basement membrane. The b4 domain mediates an interaction with both plectin and BP180; its absence prevents hemidesmosomal assembly. 53) Moving from internally to externally choose the correct description of the hair follicle: A. Henle’s layer _ Huxley’s layer _ inner root sheath cuticle _ hair shaft cuticle _ cortex _ medulla B. inner root sheath cuticle _ outer root sheath cuticle _ hair shaft cuticle _ cortex _ Huxley’s layer _ Henle’s layer C. inner root sheath cuticle _ Huxley’s layer _ Henle’s layer _ medulla _ cortex _ hair shaft cuticle D. hair shaft cuticle _ cortex _ medulla _ Henle’s layer _ Huxley’s layer _ inner root sheath cuticle E. medulla _ cortex _ hair shaft cuticle _ inner root sheath cuticle _ Huxley's layer _ Henle's layer Correct Choice (As described in correct choice) 54) Homocystinuria has abnormal crosslinking of collagen because of a mutation in: A. cystathione synthaseCorrect Choice B. lysyl oxidase C. lysyl hydroxylase D. type I collagen N-peptidase gene E. Tenascin X Homocystinuria is caused by a mutation in cystathione synthase. The main skin findings are a malar flush, livedo reticularis and leg ulcerations. A characteristic eye finding is the downward displacement of the lens. The other options are involved in abnormalities associated with Ehlers- Danlos syndrome. Lysyl hydroxylase is deficient in Kyphoscoliosis type of EDS. Tenascin X is involves in ~3% of Classical type EDS cases. Dermatosparaxis type EDS has recessive mutations in the type I collagen N-peptidase gene. 55) At what estimated gestational age are all layers of the keratinized epidermis identifiable? 15
  • 16. A. 20 weeks B. 12 weeks C. 16 weeks D. 24 weeksCorrect Choice E. 8 weeks At 24 weeks, all the layers of the mature epidermis can be identified, and the epidermis is keratinized 56) Which of the following is NOT characterized by IgG and/or C3 at the basement membrane zone? A. Mucosal pemphigoid B. Pemphigus vulgarisCorrect Choice C. Bullous pemphigoid D. Pemphigoid gestationis E. Epidermolysis bullosa acquisita All of the listed conditions except pemphigus vulgaris have IgG and/or C3 deposits at the basement membrane zone. Pemphigus vulgaris is characterized by IgG deposition in the intercellular space of the epidermis. 57) Keratinocytes have been shown to secrete all of the following cytokines except: A. IL-1 B. IL-6 C. TNF-alpha D. IL-8 E. IL-2Correct Choice Keratinocytes have been shown to secrete all of the above cytokines, except IL-2, IL-4, and IFN- gamma 58) Which one of the following is responsible for maintaining a barrier to water loss in the stratum corneum? A. involucrin B. transglutaminase C. loricrin D. odland bodiesCorrect Choice E. filaggrin Odland bodies, also known as lamellar granules, keratinosomes, and membrane-coating granules, are small organelles that are discharged from granular cells into the intracellular space of the granular layer of the epidermis. These bodies have two known functions: they mediate stratum corneum cell cohesion and they form a barrier to water loss. Odland bodies are round to oval, measure approximately 300 to 500 nm in diameter, and possess a trilaminar membrane and 16
  • 17. a laminated interior. They contain neutral sugars linked to lipids and/or proteins, hydrolytic enzymes, and free sterols. Filaggrin is a breakdown product of filaggrin precursor, a component of keratohyaline granules, which aggregates with keratin filaments and acts as a "glue" for keratin filaments. Involucrin is a cysteine-rich protein synthesized in the cytoplasm of spinous cells. The enzyme, transglutaminase, cross-links involucrin in the granular layer forming an insoluble cell boundary that is resistant to denaturing and reducing chemicals. Loricrin, is a highly insoluble sulfur- and glycine/serine-rich protein, which is the major protein comprising the cornified cell envelope. (Lever's Histopathology of the Skin. 8th ed 1997 p11) (Fitzpatrick's Dermatology in General Medicine. 5th ed 1999, p 136-138) 59) Which signaling molecule mediates the transition of hair cycling from telogen to anagen phase? A. Fgf5 B. Foxn1 C. 5a-reductase D. Dihydotestosterone E. Sonic hedgehogCorrect Choice Sonic hedgehog (Shh), a signaling molecule secreted by ectodermal cells of the developing hair follicle, appears to be critical in mediating the transition from telogen to anagen during postnatal hair cycling 60) Apocrine glands A. demonstrate decapitation secretion. Correct Choice B. are fully functional at birth. C. are thermoregulatory. D. are diffusely distributed on the body. E. demonstrate holocrine secretion. Apocrine glands show decapitation secretion. Like eccrine glands, apocrine glands are composed of three segments, the intraepidermal duct, the intradermal duct, and the secretory portion. The duct of the apocrine gland usually leads to a pilosebaceous follicle above the entrance of the sebaceous duct. Apocrine glands are found in the axillae, anogenital region, external ear canal (ceruminous glands), in the eyelids (Moll’s glands), and in the breast (mammary glands). Apocrine glans are functional only at puberty. Their initial secretion is odorless. Hidradenitis suppuritiva is a disease involving apocrine glands. 61) Which element is necessary for function of matrix metalloproteinases? A. iron B. zincCorrect Choice C. magnesium D. nitrogen E. manganese 17
  • 18. Matrix metalloproteinases are required for normal tissue architecture and normal turnover of the extracellular matric. All of them have zinc at the active site and require octahedral binding of calcium ions to maintain structural integrity. 62) The strength of a scar: A. None of these options are correct B. All of these options are correctCorrect Choice C. Is 20% at 3 weeks D. Is 5% at 1 week E. Is 70% at 1 year A scar has 5% strength at 1 week, 20% at 3 weeks and 70% at 1 year. It will never recover strength to the level of pre-injury. 63) Which hormone is homologous to alpha-MSH (melanocyte stimulating hormone)? A. insulin B. thyroid stimulating hormone C. prolactin D. adrenocorticotropic hormoneCorrect Choice E. human growth factor Both alpha-MSH and ACTH are cleavage products of proopiomelanocortin (POMC). 64) The microflora of pilosebaceous unit consist of which of the following: A. Staphylococcus aureus B. Pseudomonas aeruginosa C. Pityrosporum ovaleCorrect Choice D. Escherichia coli E. Corynebacterium diphtheriae All the above bacteria and fungi are found within sebaceous glands; the Malasssezia spp. and P. ovale are found within the acroinfundibulum, S. epidermidis is found within the midinfundibulum, and Propionibacterium spp. deep within the follicle. 65) Human sebum is distinguished from lipids of internal organs by the presence of: A. Squalene B. Wax estersCorrect Choice C. Glycerides D. Cholestrol E. Cholestrol esters As human sebum exits the sebaceous gland, its major constituents are squalene, cholesterol, cholesterol esters, triglycerides, and wax esters. With passage through the hair follicle, 18
  • 19. triyglycerides in the sebum become hydrolyzed by bacterial enzymes, so that by the time the sebum reaches the skin surface, it contains free fatty acids, mono- and diglycerides in addition to the original components. Human sebum is distinguished by the presence of wax esters and squalene. The lipids of human internal organs contain no wax esters and little squalene. The squalene that is produced in internal organs is quickly converted to lanosterol and then to cholesterol, so it does not remain in its original form. Human sebaceous glands do not convert squalene to sterols. (Fitzpatrick's Dermatology in General Medicine. 5th ed 1999, p [ ] Chapter 10 66) In X-linked ichthyosis, steroid sulfatase is missing from: A. peroxisomal granules B. lamellar granulesCorrect Choice C. basal keratinocytes D. dermal fibroblasts E. Odland bodies Steroid sulfatase is missing from lamellar granules in the stratum corneum in X-linked ichthyosis. There is an increase in cholesterol sulfate and decreases in cholesterol levels. This may be the mechanism for the retention hyperkeratosis seen in this condition. 67) Each of the following is true about the basement membrane zone except: A. Contains laminin 1 and laminin 5 B. Anchoring filaments attach the basal cell membrane to the lamin lucidaCorrect Choice C. Can be visualized on light microscopy with PAS staining D. Lamina densa is composed of type IV collagen E. Anchoring fibrils are composed of type VII collagen The basement membrane zone is seen on staining with PAS stain. It appears as a homogenous band approximately 1 micron thick at the dermo-epidermal junction. The hemidesmosomal complex and basement membrane zone play an integral role in maintaining cellular adhesion. Anchoring filaments (primarily composed of laminin 5 and BPAG2) attach the basal cell membrane to the lamina densa NOT lamina lucida. 68) Dystrophic epidermolysis bullosa results from mutations in A. collagen type III. B. collagen type II. C. collagen type I. D. collagen type VII. Correct Choice E. collagen type V. See discussion from question 12 69) Which of the following medications is concentrated in the eccrine glands? 19
  • 20. A. all of the answers are correctCorrect Choice B. cytarabine C. cephalexin D. cyclophosphamide E. ciprofloxacin The above listed drugs, as well as beta-lactam antibiotics, antifungals such as ketoconazole and griseofulvin, are known to be secreted into eccrine sweat ducts. This feature may explain the development of neutrophilic eccrine hidradenitis and eccrine squamous syringometaplasia in the context of chemotherapy. 70) What is the average duration of the telogen cycle in terminal scalp hair? A. 9 months B. 6 months C. 2-3 weeks D. 2-6 years E. 3 monthsCorrect Choice The average duration of the telogen phase of the hair cycle is 3 months; this feature explains why telogen effluvium is typically observed 3 months following a traumatic event or serious illness. The average duration of the anagen phase of the hair cycle is 2-6 years, whereas that of the catagen cycle is 2-3 weeks. 71) At any one time, the approximate proportion of hair follicles in anagen is A. 40% B. 85% Correct Choice C. 95% D. 60% E. 15% Most hair follicles are in anagen, and thus most hair follicles involve growing hair. The longer a hair follicle is anagen, the longer the hair can grow in length. Hairs of the scalp grow approximately 0.4 mm per day, and thus the date of your next hair cut can be accurately calculated. 72) Desmosine and isodesmosine are typical amino acids found in: A. Collagen fibers B. Anchoring plaques C. Anchoring fibril D. Elastic fibersCorrect Choice E. Heparan sulfate Desmosine and isodesmosine are typical amino acids found in elastic fibers. They crosslink fibrillin. Anchoring fibrils are composed of collagen VII and collagen fibers and have the most 20
  • 21. typical amino acids of proline and hydroxyproline. Heparan sulfate do not typically contain these amino acids. 73) What is the location of the unbound corticosteroid receptor? A. mitochondria B. golgi apparatus C. nucleus D. plasma membrane E. cytoplasmCorrect Choice Both androgen and corticosteroid receptors localize to the cytoplasm. Estrogen receptors are found in the nucleus. Progesterone receptors are distributed in both the nucleus and the cytoplasm 74) The epidermis is comprised of what type of cells? A. Keratinocytes, Melanocytes, Neutrophils, Langerhan cells B. Keratinocytes, Endothelial cells, Merkel cells, Goblet cells C. Keratinocytes, Melanocytes, Merkel cells, Goblet cells D. Keratinocytes, Endothelial cells, Merkel cells, Langerhan cells E. Keratinocytes, Melanocytes, Merkel cells, Langerhan cellsCorrect Choice The adult epidermis is composed of three basic cell types: Keratinocytes, melanocytes, and Langerhans cells. An additional cell, the Merkel cell, can be found in the basal layer of the palms and soles, the oral and genital mucosa, the nail bed, and the follicular infundibula. 75) During embryogenesis, periderm cells of the fetus contain which of the following substances? A. Ceramide B. Sebum C. GlycogenCorrect Choice D. Free fatty acids E. Porphyrins In week 7 of embryogenesis the surface ectoderm produces two layers. The external layer is the periderm which contains glycogen and gives rise to the stratum corneum by week 21. The other layer is the stratum germinativum. 76) The desmosomal connections of the epidermis are dependent on which of the following ions? A. CalciumCorrect Choice B. Iron C. Zinc D. Sodium E. Selenium 21
  • 22. The desmosomal connections in the epidermis are calcium dependent. The other options are not required for these connections. 77) Surgery, Parturition, Fever, Kwashiokor and Hypervitaminosis A are all causes of: A. Telogen effluviumCorrect Choice B. Alopecia areata C. Both telogen and anagen effluvium D. Anagen effluvium E. Androgenetic alopecia All of the listed stressors can induce telogen effluvium. Other causes include traction and some drug exposures. Usually will only involve up to 50% of scalp hairs and will resolve within 2-3 months 78) Anchoring fibrils are primarily composed of A. Type II collagen B. Type IV collagen C. Type III collagen D. Type VII collagen Correct Choice E. Type I collagen Anchoring fibrils are found in the sublamina densa and are made up of collagen type VII. This collagen type is mutated in dystrophic epidermolysis bullosa, and targeted in epidermolysis bullosa acquisita and bullous lupus erythematosus 79) Which eponym describes vestigial lines of pigmentary demarcation? A. dermatome B. Langer's lines C. lines of Blaschko D. Fuchter linesCorrect Choice E. Wallace's lines Fuchter lines are vestigial lines in which the dorsal surface has more melanocytes than ventral surface. Wallace's lines are the well-demarcated lines around the margin of the foot and hand. 80) Elaunin fibers A. run parallel in bands within the superficial papillary dermis. B. run parallel in bands within the reticular dermis. Correct Choice C. run perpendicular from the dermo-epidermal junction within the superficial papillary dermis. D. run perpendicular in bands within the deep dermis. E. run perpendicular in bands within the reticular dermis. 22
  • 23. Elaunin fibers are elastic fibers that have less elastin and more fibrillin and run parallel in thin bands within the reticular dermis. Oxytalin fibers contain no elastin and run perpendicular from the dermo-epidermal junction within the superficial papillary dermis. Elastic fibers turn over slowly in the skin, and are damaged by ultraviolet radiation. 81) Sebaceous glands secrete sebum through which of the following secretory mechanisms? A. Holocrine and Merocrine B. Merocrine C. Holocrine and Apocrine D. HolocrineCorrect Choice E. Apocrine Sebaceous glands exhibit holocrine secretion, whereby the sebocytes disintegrate in transit to the gland center, releasing their sebum contents. Merocrine secretion refers to the formation of intracellular secretory vesicles that translocate to the apical cell surface for secretion. Apocrine secretion refers to the process whereby secretory contents are packaged using the apical cell membrane, and 'pinched off' to achieve secretion. 82) The cutaneous immunofluorescence pattern in patients with Senear-Usher syndrome is: A. intercellular IgG on guinea pig esophagus B. intercellular IgG and C3 and linear IgG and C3 along the basement membrane zoneCorrect Choice C. linear IgG and C3 along the basement membrane zone D. linear IgG and C3 along the basement membrane zone and intercellular IgG on guinea pig esophagus E. intercellular IgG and C3 Senear-Usher syndrome, or pemphigus erythematosus, is a variant of pemphigus foliaceus characterized by crusted papules and plaques with a seborrheic distribution (malar region, scalp, upper chest and back). They demonstrate in-vivo IgG and C3 deposition on keratinocyte cell membranes and the basement membrane zone, in addition to circulating anti-nuclear antibodies. 83) Sebaceous glands secrete sebum via A. Exostosis B. Endocrine mechanism C. Exocrine mechanism D. Holocrine mechanismCorrect Choice E. Mecrocrine mechanism The sebaceous lobules have basal germinative cells and central sebocytes, which gradually become more distended with lipid vacuoles until they are shed into the lumen (holocrine secretion). 84) Which of the following is not a specialized type of sebaceous gland? 23
  • 24. A. Fordyce spot B. Montgomery's tubercle C. Meibomian gland D. Gland of Zeis E. Moll's glandCorrect Choice There are several types of specialized sebaceous glands that are not associated with a hair follicle. They include Montgomery's areolar tubercle, Fordyce spots of the lip, Glands of Zeis of the cutaneous eyelid, and Meibomian glands of the eyelid. Moll's gland of the eyelid are a modified apocrine gland. 85) Keratohyalin granules contain A. Envoplakin and Keratin 6 B. Numerous Golgi apparati C. Desmoplakin D. Profilaggrin and loricrin Correct Choice E. Involucrin Keratohyalin granules are found in the stratum granulosum (the granular layer), and contain the proteins profilaggrin and loricrin. Profilaggrin is converted to filaggrin during the transformation of the granular layer to the cornified layer. This is a calcium-dependent process. Loricrin comprises 75% of the cornified envelope’s mass. 86) Which cadherin is responsible for adhesion of Langerhan cells to the epidermis? A. Desmocollin B. E-cadherinCorrect Choice C. P-cadherin D. N-cadherin E. Desmoglein E-cadherin is responsible for the adhesion of Langerhan cells to the epidermis. There are two major subclasses of cadherins which mediate cell adhesion and play a fundamental role in normal development, classic (E-,P-,N-cadherin) and desmosomal (desmoglein and desmocollin). They depend on calcium for their function. 87) Anagen effluvium is best described as A. a cessation of mitotic activity in rapidly dividing hair matrix cells Correct Choice B. a scarring alopecia affecting only anagen stage follicles C. a cessation of mitotic activity in resting hair matrix cells D. an abrupt transition of telogen to anagen in resting hair matrix cells E. an abrubt transition from anagen to catagen in rapidly dividing hair matrix cells Anagen effluvium results from an outside stimulus – most often an antimetabolite, chemotherapeutic drug – inducing an abrupt cessation of hair matrix cell mitotic activity. This 24
  • 25. process occurs within days to weeks of the stimulus, and is reversible with cessation of the drug therapy. 88) Retinoids upregulate transcription of which types of collagen? A. 1 and 7Correct Choice B. 3 and 7 C. 4 and 7 D. 1 and 3 E. 1 and 4 Retinoids upregulate the transcription of collagens one and seven thereby strengthening the dermis 89) In the epidermis, the cell most responsible for antigen detection and processing is the A. Melanocyte B. Merkel Cell C. Langerhans cell Correct Choice D. CD4+ T cell E. Keratinocytes The Langerhans cell is a bone-derived, antigen-presenting cell found in all layers of the epidermis, oral mucosa, esophagus, and vagina. Langerhans cells ingest and process antigens, mature, migrate to a local lymph node, and then present the antigen to a naïve (or resting) T cell, activating that T cell. The Langerhans cell is central to the pathogenesis of atopic dermatitis, psoriasis, allergic contact dermatitis, and certain infections, such as Leishmaniasis. 90) The formation of granulation tissue depends on the presence of A. platelets. B. collagen type I. C. collagen type IV. D. fibronectin. Correct Choice E. neutrophils. See discussion of wound healing from questions 29 through 33. 91) People with darker skin show A. a grouping of melanosomes with a low degree of melanization B. a lessened production of melanosomes within melanocytes C. a higher degree of dispersion of melanosomes in keratinocytes Correct Choice D. smaller, more concentrated melanosomes E. a more rapid degradation of melanosomes 25
  • 26. More darkly pigmented races show a greater production of melanosomes in the melanocyte, melanosomes with a higher degree of melanization, larger melanosomes, a higher degree of dispersion of melanosomes in the keratinocytes, and a slower rate of melanosome degradation. 92) Which of the following enzymes does not require copper for functioning? A. tyrosinase B. cystathione beta-synthase C. ferrochelataseCorrect Choice D. lysyl oxidase E. ATP7a All of the listed enzymes are copper containing or dependent except ferrochelatase. Lysyl oxidase facilitates crosslinking of fibrillin in elastic fibers. ATP7a is deficient in Menkes Kinky Hair Syndrome. Cystathione beta-synthase is defective in homocystinuria. Tyrosinase catalyzes the first 2 steps, and at least 1 subsequent step, in the conversion of tyrosine to melanin. Ferrochelatase mutation leads to excess protoporphyrin production and photosensitivity. 93) When do melanocytes begin to synthesize melanin? A. 2nd month of gestation B. 4th month of gestation C. 3rd month of gestationCorrect Choice D. 6th month of gestation E. 5th month of gestation Melanocytes begin to synthesize melanin in the 3rd month of gestation. 94) Red or blonde hair pigmentation primarily results from A. the reduced activity of DOPA dehydroxylase. B. the presence of eumelanin. C. the presence of pheomelanin. Correct Choice D. the reduced activity of tyrosinase. E. the absence of melanin. Hair color is determined by melanocytes. The melanocytic activity of follicular melanocytes is coupled to anagen – hair is only pigmented when it is growing. Pigment is produced in the matrix area of follicle, above the follicular papilla. Eumelanin is the pigment of brown/black hairs, and pheomelanin is the pigment of red/blonde hairs. Intensity of color is proportional to the amount of pigment. The absence of pigment produces white hair, and markedly reduced pigment produces gray hair. 95) Sebaceous glands A. respond to adrenergic neural activity, exclusively. B. respond to the local release of cytokines from inflammatory cells. 26
  • 27. C. respond to chemical stimuli such as hormones. Correct Choice D. respond to cholinergic neural activity, exclusively. E. respond to both adrenergic and cholinergic stimuli. Sebaceous glands are androgen-responsive holocrine glands that enlarge at puberty. Meibomian glands of the eyelids are modified sebaceous glands. Sebaceous glands are found everywhere on the skin except the palms and soles. Fordyce’s condition involves free sebaceous glands on the vermillion border of the lips and on the buccal mucosa. Eccrine glands are thermoregulatory structures that respond to cholinergic stimulation. 96) Anchoring filaments originate at the hemidesmosomes and insert into the A. Sub basal dense plate B. BPAG 180 C. Lamina lucida D. Lamina densaCorrect Choice E. Desmosome Anchoring filaments (smaller than anchoring fibrils) stretch from the plasma membrane through the subbasal dense plaque and the lamina lucida to the lamina densa 97) Keratinocytes are derived from which of the following: A. Endoderm B. Neural Crest C. Bone marrow precursors D. EctodermCorrect Choice E. Mesoderm As implied by the root 'ecto', a prefix meaning "outer", the keratinocytes of the epidermis are derived from the ectoderm. The other layers do contribute cell populations that are present in the skin. 27